Vous êtes sur la page 1sur 52

JEE (MAIN & ADV.

), MEDICAL + BOARD, NDA, X & IX


Enjoys unparalleled reputation for best results
in terms of percentage selection
www.newtonclasses.net

STATISTICS ( )
Only one option is correct.
1. Which of the following is not a measure of central tendency ?
(a) mean (b) mode (c) mean – deviations (d) median
2. The class marks of a distribution are 6, 10, 14, 18, 22, 26, 30 then the class size is
(a) 2 (b) 4 (c) 5 (d) 8
3. The mean of 10 observations is 16.3. By an error one observation is registered as 32 instead of 23.
Then, the correct mean is
(a) 15.4 (b) 15.6 (c) 15.7 (d) 15.8
4. The A. M of 1,3, 32 ,.....3n−1 is
3n −1 3n − 3 3n −1 − 1 3n +1 − 1
(a) (b) (c) (d)
2n 2n 2n 2n
5. For the given frequency distribution the mean is 12.58. find x1 ,
xi 5 8 10 12 x1 20 25

fi 2 5 8 22 7 4 2
(a) 13 (b) 15 (c) 17 (d) 19
6. If the average of the numbers 1, 2, 3,…. , 98, 99, x is 100x , then the value of x is
50 51 50 51
(a) (b) (c) (d)
99 99 101 100
7. For the given frequency distribution, the mean is 7.68. find the value of the missing frequency f1 .
xi 3 5 7 9 11 13

fi 6 8 15 f1 8 4
(a) 7 (b) 9 (c) 11 (d) 13
8. If the mean of the numbers 27 + x,31 + x,89 + x,107 + x,156 + x is 82, then the mean of
130 + x,126 + x, 68 + x,50 + x,1 + x is
(a) 75 (b) 79 (c) 80 (d) 82
9. The Number which is mean of the squares of deviations from mean called ………

(a) Standard deviation (b) variance (c) median (d) None of these
10. The A.M of 9 terms is 15. If one more term is added to this series, then the A.M. becomes 16. The
value of the added term is
(a) 23 (b) 25 (c) 27 (d) 29
11. Mean of 100 items is 49. It was discovered that 3 items 60, 70, 80 were wrongly read as 38, 22, 50
respectively. The correct mean is
(a) 48 (b) 50 (c) 78 (d) 80
12. In the following table, the frequencies f1 and f 2 corresponding to the classes 20 – 40 and 60 – 80 are
missing. The mean of the given distribution is 50. The value of the greater of the two frequencies
f1 and f 2 is

th
Office.: 606 , 6 Floor, Hariom Tower, Circular Road, Ranchi-1, Ph.: 0651-2562523, 9835508812, 8507613968
2 ( ) BY R. K. MALIK’S NEWTON CLASSES
Class 0 – 20 20 – 40 40 – 60 60 -80 80 - 100 ∑ f

Frequency 17 f1 32 f2 19 120
(a) 23 (b) 24 (c) 28 (d) 29
13. The arithmetic mean of 7 consecutive integers starting with a is m . Then, the arithmetic mean of 11
consecutive integers starting with a + 2 is
(a) 2a (b) m + 2 (c) m + 4 (d) 9 + m + 2
14. The sum of the squares of deviations from mean is
(a) 0 (b) least (c) maximum (d) none of these
15. On dividing each entry in a data by a non – zero number ' a ' , the arithmetic mean of the new data
(a) does not change (b) is multiplied by a (c) is diminished by a (d) is divided by a
16. Mean of 12 numbers is 26. If 5 is added in each number , then new mean is
(a) 25 (b) 31 (c) 60 (d) 84
17. The mean of four observations is 20. when a constant C is added to each observation, the mean
becomes 22. The value of C is
(a) −2 (b) 2 (c) 4 (d) 6
18. The mean marks obtained in a subject A by a class of 10 students is 66. Two weakest students who
scored 34 and 37 marks respectively were made to leave the class. A new students having 52 marks
took admission to this class. The mean marks of the students of the new class so formed, are
(a) 69.23 (b) 71.22 (c) 74.47 (d) 78.33
19. The average weight of students in a class of 35 students is 40 kg. If the weight of the teacher be
1
Included, the average rises by kg. The weight of the teacher is
2
(a) 42 kg (b) 46 kg (c) 52 kg (d) 58 kg
20. The mean marks in statistics of 100 students in a class was 72. The mean marks of boys was 75, while
their number was 70. The mean marks of girls in the class was
(a) 60 (b) 62 (c) 65 (d) 66
21. A.M. of a set of 50 numbers is 38. Two numbers are 45 and 55. The A. M. of the rest of the numbers is
(a) 36 (b) 36.5 (c) 37 (d) 37.5
22. The average monthly salary of workers in a factory is Rs.206 . If the average monthly salary of males
and females are Rs. 210 and Rs. 190 respectively, then the percentage of females employed in the
factory is
1
(a) 20% (b) 25% (c) 30% (d) 33 %
3
23. The median of 1, 10, 3, 9, 4, 8, 5 is
(a) 4 (b) 3.5 (c) 5 (d) 6.5
24. Find the median of 18, 35, 10, 42, 21.
(a) 19 (b) 20 (c) 21 (d) 22
1 5
25. The median of the observations 3, 8, 11, 16, x − , x + , 29, 35, 37, 41 arranged in an ascending
2 2
order is 23. The value of x is
1 1
(a) 20 (b) 21 (c) 21 (d) 22
2 2
26. The observation which occurs most frequently is known as
(a) mean (b) weighted mean (c) median (d) mode
27. Mode of 7, 6, 10, 7, 5, 9, 3, 7, 5 is
(a) 3 (b) 5 (c) 6 (d) 7

th
Office.: 606 , 6 Floor, Hariom Tower, Circular Road, Ranchi-1, Ph.: 0651-2562523, 9835508812, 8507613968
(BY R. K. MALIK’S NEWTON CLASSES
) 3
28. The mode of the given distribution is
Weight ( in kg ) 40 43 46 49 52 55

Number of children 5 8 16 9 7 3
29. Find the mode of the data given below :
Marks 0-5 5- 10 10 - 15 15 - 20 20 – 25 25- 30
Obtained
Number of 18 20 25 30 16 14
students
30. Mean and mode of a data are 66 and 60, median is
(a) 19 (b) 28 (c) 32 (d) 64
31. If in a frequency distribution, the mean and median are 21 and 22 respectively, then its mode is
approximately
(a) 20.5 (b) 22.0 (c) 24.0 (d) 25.5
32. For a moderately skewed distribution, mean = 34, median = 36 , then the mode is
(a) 30 (b) 35 (c) 40 (d) 45
33. The mean deviation from the mean of the set of observations −1, 0, 4 is
(a) 1 (b) 2 (c) 3 (d) 4
34. Mean deviation of 6, 8, 12, 15, 10, 9 through mean is
(a) 2 (b) 2.33 (c) 4.52 (d) 10
35. The mean deviation about the mean for the values 18, 20, 12, 14, 19, 22, 26, 16, 19, 24 is
(a) 3.1 (b) 3.2 (c) 3.3 (d) 3.4
36. The runs scored by a batsman in 10 innings are 38, 70, 48, 34, 42, 55, 63, 46, 54 and 44. The mean
deviation is
(a) 6.44 (b) 7.64 (c) 8.88 (d) 9.42
37. If V is the variance and σ is the standard deviation, then
1 1
(a) V = (b) V = 2 (c) V = σ 2 (d) V 2 = σ
σ σ
38. The variance of 35, 40, 42, 36, 27 is
(a) 25.4 (b) 25.9 (c) 26.8 (d) 27.2
39. The variance of the data 2, 4, 6, 8, 10 is
(a) 6 (b) 7 (c) 8 (d) 9
40. The standard deviation for the data 7, 9, 11, 13, 15 is
(a) 2.4 (b) 2.5 (c) 2.7 (d) 2.8
41. The standard deviation of 25 numbers is 40. If each of the numbers increases by 5, then the standard
deviation will be
21
(a) 40 + (b) 40 (c) 45 (d) 200
25
42. If x is a random variable with values x1 = 4, x2 = −3 , x3 = 6 and x4 = 1 , then the standard deviation of
x equals
(a) 2 (b) 5.5 (c) 11.5 (d) none of these
43. The variance of 20 observations is 10. if each observation is multiplied by 3, the new variance of the
resulting set of observations is
(a) 9 (b) 30 (c) 90 (d) 300
44. The standard deviation for the scores 1, 2, 3, 4, 5, 6 and 7 is 2. Then, the standard deviation of 12, 23,
34, 45, 56, 67 and 78 is

th
Office.: 606 , 6 Floor, Hariom Tower, Circular Road, Ranchi-1, Ph.: 0651-2562523, 9835508812, 8507613968
4 ( ) BY R. K. MALIK’S NEWTON CLASSES
(a) 11 (b) 13 (c) 22 (d) 23
n n
45. The standard deviation of n observations x1 , x2 ,....., xn is 2. If ∑ xi = 20 and
i =1
∑x
i =1
2
i = 100 , then n is

(a) 5 or 15 (b) 5 or 20 (c) 10 or 15 (d) 10 or 20


46. The mean of five numbers is 0 and their variance is 2. If three of those numbers are −1,1 and 2, then
the other two numbers are
(a) −2 and 0 (b) −3 and 1 (c) −4 and 2 (d) −5 and 3
47. The mean and variance of six observations are 8 and 16 respectively. If each observation is multiplied
by 3, then the new mean and the new variance of the resulting observations are
(a) 24, 48 (b) 11, 48 (c) 12, 24 (d) 24, 144
48. What is the standard deviation of the following data:
Measurements 0 - 10 10- 20 20 - 30 30 - 40

Frequency 1 3 4 2
(a) 2.26 (b) 7.6 (c) 9.0 (d) 81.0
49. The mean and variance of seven observations are 8 and 16 respectively. If five of these are 2, 4, 10, 12
and 14 , then the remaining two are
(a) 6, 8 (b) 7, 11 (c) 8, 13 (d) 9, 15
50. For a sequence of 9 observations, the arithmetic mean and the standard deviation are 43 and 5
respectively. If a new observation of value 63 is added to this sequence, then the standard deviation of
the series so formed is
(a) 6.15 (b) 6.85 (c) 7.35 (d) 7.65
51. If the mean of five observations x, x + 2, x + 4, x + 6 and x + 8 is 11, then the mean of last three
observations is
(a) 11 (b) 13 (c) 15 (d) 17
52. The median from the table is
x 7 8 10 9 11 12 13
f 2 1 4 5 6 1 3

(a) 9 (b) 12 (c) 11 (d) 10


53. Mean of 10 numbers is 6. It was later observed that one number was misread as 9. If the correct mean
was 7, what is the correct value of that number ?
(a) 19 (b) 20 (c) 8 (d) 10
54. A market with 3900 operating firms has the following distribution for firms arranged according to
various income groups of workers.
Income group No. of firms

150 – 300 300

300 – 500 500

500 – 800 900

800 - 1200 1000

1200 – 1800 1200

If a histogram for the above distribution is constructed the highest bar in the histogram would
correspond to the class :

th
Office.: 606 , 6 Floor, Hariom Tower, Circular Road, Ranchi-1, Ph.: 0651-2562523, 9835508812, 8507613968
( BY R. K. MALIK’S NEWTON CLASSES
) 5
(a) 500 - 800 (b) 1200 -1800 (c) 800 -1200 (d) 150 – 300
55. The quartile deviation of the distribution is :

x 2 3 4 5 6
y 2 4 8 4 1

(a) 0 (b) 1 (c) 1/ 4 (d) 1/ 2


56. The mean of 10 values of a group is 6. If among these values, the mean of 4 values is 7.5, then the
mean of the remaining value s will be :
(a) 4.5 (b) 5 (c) 5.5 (d) 6.5
57. The quartile deviation of 12, 7,15,10,17,19 , 25 is :
(a) 4.5 (b) 5 (c) 9 (d) 14.5
58. If the mean of 7 observations of group is 10 and that of 3 observations of another group is 5, then the
mean of the combined group will be
(a) 10 (b) 15 (c) 8.5 (d) none of these
59. The standard deviation of the following distribution
x 0 – 10 10 – 20 20 – 30 30 – 40
f 1 3 4 2
(a) 9 (b) 7.6 (c) 8.1 (d) 2.26
60. The mean of observations x1 , x2 ,....., xn is x . If the observation x p is replaced by x′p , then the new
mean will be
n x − x p + x′p ( n − 1) x + x′p
(a) x − x p + x′p (b) (c) (d) none of these
n n
61. In a class of 100 students, there are 70 boys whose average marks are 750. If the average marks of the
complete class are 720, then the average marks of the girls are
(a) 700 (b) 650 (c) 690 (d) 680
62. If the mean of 3, 4, x , 7, 10 is 6, then the value of x is
(a) 4 (b) 5 (c) 6 (d) 7
63. The median of the data 13, 14, 16, 18, 20, 22 is
(a) 17 (b) 16 (c) 18 (d) none of these
64. The mode of the following items is 0, 1, 6, 7, 2, 3, 7, 6, 6, 2, 6, 0, 5, 6, 0
(a) 0 (b) 5 (c) 6 (d) 2
65. Consider the frequency distribution of the given numbers

Value 1 2 3 4

Frequency 5 4 6 f

If the mean is known to be 3, then the value of f is


(a) 3 (b) 7 (c) 10 (d) 14
66. The mean of 5 numbers is 18. If one number is excluded, their mean becomes 16. Then the excluded
number is
(a) 18 (b) 25 (c) 26 (d) 30
67. If the standard deviation of 1, 2, 3, 4, ….10 is σ , then the standard deviation of 11, 12, 13, 14,…,20 is
(a) σ + 10 (b) 10 σ (c) σ (d) none of these
68. The standard deviation of 25 numbers is 40. If each of the numbers is increased by 5, then the new
standard deviation will be
21
(a) 40 (b) 45 (c) 40 + (d) none of these
25
1
69. If x1 , x2 , x3 ,....xn be n observations, then the quantity ( x1 , x2 , x3 ,....xn ) n is called
(a) H.M (b) A.M (c) G.M (d) none of these

th
Office.: 606 , 6 Floor, Hariom Tower, Circular Road, Ranchi-1, Ph.: 0651-2562523, 9835508812, 8507613968
6 ( BY R. K. MALIK’S NEWTON CLASSES
)
70. The mean weight per student in a group of seven students is 55 kg if the individual weights of 6
students are 52, 58, 55, 53, 56 and 54 ; then weights of the seventh student is
(a) 55 kg (b) 60 kg (c) 57 kg (d) 50 kg
71. If Mean = ( 3 Median − Mode ) x , then the value of x is
(a) 1 (b) 2 (c) 1/ 2 (d) 3 / 2
72. S.D. of a data is 6. When each observation is increased by 1, then the S.D. of new data is
(a) 5 (b) 7 (c) 6 (d) 8
73. The S.D. of 15 items is 6 and if each item is decreased by 1, then standard deviation will be
(a) 5 (b) 7 (c) 91/15 (d) 6
74. Consider the following data which represents the runs scored by two batsmen in their last ten matches
as
Batsman A : 30, 91, 0, 64, 42, 80, 30, 5, 117, 71 Batsman B: 53,46,48,50, 53, 53,58, 60, 57, 52
Which of the following is / are true about the data ?
(i) Mean of batsman A runs is 53 (ii) Median of batsman A runs is 42
(iii) Mean of batsman B runs is 53 (iv) Median of batsman B runs is 53.
(a) Only I is true (b) I and III are true (c) I ,III and IV are true (d) All are true
75. Consider the following statements :
(I) Measures of dispersion Range , Quartile deviation ,mean deviation variance , standard deviation are
measures of dispersion
Range = Maximum value – minimum values
(II) Mean deviation form mean and median are for ungrouped data

∑ x −x 1 ∑ x −M
1

()
M .D x =
n
, M .D ( M ) =
n
respectively

(III) Mean deviation from mean and median are for grouped data

∑f 1 x1 − x ∑f 1 x1 − M
()
M .D x =
N
, M .D ( M ) =
N
respectively

Where N = ∑ f1

Which of the above statements are true ?


(a) Only (I) (b) Only (II) (c) Only (III) (d) All of the above
76. The range of data 9,8,12,80,75,45, 36, 72 is
(a) 9 (b) 80 (c) 72 (d) 75
77. The range of the following set of observations 2, 3, 5, 9, 8, 7, 6, 5, 7, 4, 3, is
(a) 11 (b) 7 (c) 5.5 (d) 6
78. If mean of 3, 4, x, 7, 10, is 6, then the value of x is
(a) 4 (b) 5 (c) 6 (d) 7
79. The mean of mean value and median value of the odd divisors of 360 is
(a) 13 (b) 7 (c) 6 (d) 10
80. The mean of a set of numbers is x . If each item is decreased by 4, then mean of new observations (set)
is
(a) unaltered (b) raised by 4 units in x (c) reduced by 4 units in x (d) None of these
81. The mean of the first ( n + 1) natural numbers is

th
Office.: 606 , 6 Floor, Hariom Tower, Circular Road, Ranchi-1, Ph.: 0651-2562523, 9835508812, 8507613968
( ) BY R. K. MALIK’S NEWTON CLASSES 7
2

(a)
( n + 2) (b)
( n + 1)( n + 2 )  n ( n + 1) 
(c)  (d)
( n + 1)

2 2  2  2
82. If x1 , x2 , x3 ,...., xn is the set of n observations whose mean is x , then
n n n n

∑ ( xi − x ) ≥ 0 (b) ∑ ( xi − x ) = 0 ∑ ( xi − x ) = 0 ∑( x − x )
2
(a) (c) (d) i =0
i =1 i =1 i =1 i =1

83. The mean of the given data is 30.


Class 0-10 10-20 20-30 30-40 40-50
Frequency 8 10 f1 15 f2
If total data is 70, then missing numbers are
(a) 14, 23 (b) 25, 21 (c) 24, 13 (d) 40, 31
84. If the mean of the distribution
Variate 6 7 1.9 9 10
Frequency 1 2 k 3 4
is 3.6. The value of k =
(a) 30 (b) 12 (c) 10 (d) 11
85. If A.M. of x1 , x2 ,....xn is x , then A.M. of ax1 + b, ax2 + b, ax3 + b,...., axn + b is
(a) ax (b) ax + b (c) ax + nb (d) None of these
86. The mean of following frequency table is 50.
Class Frequency
0-20 17
20-40 f1
40-60 32
60-80 f2
80-100 19
Total 120
The missing frequencies are
(a) 28, 24 (b) 22, 36 (c) 36, 28 (d) 35, 17
87. The average weight of 25 boys was calculated to be 78.4 kg. It was later discovered that one weight
was misread as 69 kg instead of 96 kg. The correct average is
(a) 79 kg (b) 79.48 kg (c) 81.32 kg (d) 80.04 kg
88. If the mean of number 27 + x,31 + x,89 + x, 107 + x,156 + x is 82, then the mean of
130 + x,126 + x, 68 + x,50 + x,1 + x is
(a) 79 (b) 75 (c) 82 (d) 80
89. The average weight of students in a class of 35 students is 40 kg. If the weight of the teacher be
included, the average rises by 1/2 kg. The weight of the teacher is
(a) 40.5 kg (b) 50 kg (c) 41 kg (d) 58 kg
90. Mean marks scored by the students of a class is 53. The mean marks of the girls is 55 and the mean
marks of the boys is 50. What is the percentage of girls in the class?
(a) 60% (b) 40% (c) 50% (d) 45%
91. The average monthly salary of workers in a factory is Rs. 206. If the average monthly salary of males
and females are Rs. 210 and Rs. 190 respectively, the percentage of female employed in the factory is
(a) 10% (b) 50% (c) 30% (d) 20%
92. The arithmetic mean of 7 consecutive integers starting with a is m. Then the arithmetic mean of 11

th
Office.: 606 , 6 Floor, Hariom Tower, Circular Road, Ranchi-1, Ph.: 0651-2562523, 9835508812, 8507613968
8 ( ) BY R. K. MALIK’S NEWTON CLASSES
consecutive integers starting with a + 2 is
(a) 2a (b) 2m (c) a + 4 (d) m + 4
93. The A.M. of 9 items is 15. If one more term is added to this data, then the A.M. becomes 16. The value
of the added term is
(a) 30 (b) 27 (c) 25 (d) 23
94. If the average of the numbers 1, 2, 3,….98, 99, x is 100x, then the value of x is
51 50 50 51
(a) (b) (c) (d)
100 99 101 99
th
95. A batsman in his 16 inning makes a score of 70 runs, and thereby increases his average by 2 runs. If
he had never been ‘not out’, then his average after 16th innings is
(a) 36 (b) 38 (c) 40 (d) 42
96. Find the range of the data 35, 50, 48, 62, 27, 39, 43, 72, 56, 68.
(a) 25 (b) 27 (c) 45 (d) 39
97. Which of the following statements is/are true?
I. The values of median and mode can be determined graphically.
II. Mean, median and mode have the same unit. III. Range is the best measure of dispersion
(a) Both I and II (b) Only II (c) Both II and III (d) None of these
98. Find the mean deviation about the median for the data give below:
11, 3, 8, 7, 5, 14, 10, 2, 9
(a) 3 (b) 2 (c) 5 (d) 4
99. Calculate the mean deviation about the median for the following data
Height (in cm) 95-105 105-115 115-125 125-135 135-145 145-155
Number of boys 9 13 25 30 13 10
(a) 17.34 (b) 12.34 (c) 15.33 (d) 11.34
100. Calculate the mean deviation about the median for the following data
Class 16-20 21-25 26-30 31-35 36-40 41-45 46-50 51-55
Frequency 5 6 12 14 26 12 16 9
(a) 9.35 (b) 7.35 (c) 26 (d) 14.35
101. Find the mean deviation about the mean for the following data
6, 7, 10, 12, 13, 4, 8, 12
(a) 2.75 (b) 1.75 (c) 3 (d) 2
102. Find the mean deviation about the mean for the following data
12, 3, 18, 17, 4, 9, 17, 19, 20, 15, 8, 17, 2, 3, 16, 11, 3, 1, 0, 5
(a) 8.2 (b) 9.2 (c) 6.2 (d) 5.2
103. Find the mean deviation about the median for the following data
3, 9, 5, 3, 12, 10, 18, 4, 7, 19, 21
(a) 5.27 (b) 6.29 (c) 4.27 (d) 3.29
104. Find the mean deviation about the mean for the following data
xi 2 5 6 8 10 12
fi 2 8 10 7 8 5
(a) 3.3 (b) 6.3 (c) 2.3 (d) 5
105. Find the mean deviation about the median for the following data
xi 3 6 9 12 13 15 21 22
fi 3 4 5 2 4 5 4 3

th
Office.: 606 , 6 Floor, Hariom Tower, Circular Road, Ranchi-1, Ph.: 0651-2562523, 9835508812, 8507613968
BY R. K. MALIK’S NEWTON CLASSES
( ) 9
(a) 4.97 (b) 5.9 (c) 2.99 (d) 3.97
106. Find the mean deviation about the mean for the following data
Marks obtained 10-20 20-30 30-40 40-50 50-60 60-70 70-80
Number of students 2 3 8 14 8 3 2
(a) 12 (b) 10 (c) 11 (d) 9
107. Calculate the mean deviation about median for the following data

Class 0-10 10-20 20-30 30-40 40-50 50-60


Frequency 6 7 15 16 4 2
(a) 10.16 (b) 11.12 (c) 12.16 (d) 9.16
108. The mean deviation of items x, x + y, x + 2 y,......, x + 2ny from mean is
n ( n + 1) y ( n + 1) y ny ( 2n + 1) y
(a) (b) (c) (d)
2n + 1 2n + 1 2n + 1 n ( n + 1)
109. Find the mean deviation about the mean of the following data
Size (x) 1 3 5 7 9 11 13 15
Frequency (f) 3 3 4 14 7 4 3 4
(a) 2.95 (b) 2.35 (c) 3.05 (d) 3.67
110. The mean deviation of the data 2, 9, 9, 3, 6, 9, 4 from the mean is
(a) 2.23 (b) 2.57 (c) 3.23 (d) 3.57
111. Find the mean derivation about the mean for the following data
xi 1 4 9 12 13 14 21 22
fi 3 4 5 2 4 5 4 3
(a) 5.33 (b) 4.33 (c) 6.33 (d) 8
112. Calculate the mean deviation form the mean of the following data
Marks 0-10 10-20 20-30 30-40 40-50 50-60 60-70
Number of students 4 6 10 20 10 6 4
(a) 12.33 (b) 11.33 (c) 20 (d) 13
113. The scores of batsman A in 10 different test matches were 38, 70, 48, 34, 42, 55, 63, 46, 54, 44. Find
the mean deviation about median.
(a) 8.6 (b) 10.61 (c) 6.8 (d) 9.61
114. The mean of five observations is 4 and their variance is 5.2. If three of them are 1, 2, 6, then other two
are
(a) 2, 9 (b) 4, 7 (c) 5, 6 (d) 2, 10
115. Find the mean of first 10 multiples of 3.
(a) 15.5 (b) 17.5 (c) 16.5 (d) 18.5
116. Find the variance of first 10 multiples of 3.
(a) 70.15 (b) 74.15 (c) 73.15 (d) 74.25
117. Given
xi 10 15 18 20 25
fi 3 2 5 8 2
Find the variance.
(a) 19 (b) 17 (c) 18 (d) 16
Directions (118 - 120) : Given below are the diameters of circles (in mm) drawn in a design.

th
Office.: 606 , 6 Floor, Hariom Tower, Circular Road, Ranchi-1, Ph.: 0651-2562523, 9835508812, 8507613968
10 ( BY R. K. MALIK’S NEWTON CLASSES
)
Diameter 33-36 37-40 41-44 45-48 49-52
Number of circles 15 17 21 22 25
118. Find the mean diameter of the circles.
(a) 43.5 (b) 42.5 (c) 45.5 (d) 44.5
119. Find the variance.
(a) 30.84 (b) 33.84 (c) 40.84 (d) 42.84
120. Find the standard deviation
(a) 4.55 (b) 3.55 (c) 5.55 (d) 2.55
121. The variance of 20 observation is 5. If each observation is multiplied by 2, find the new variance of the
resulting observations
(a) 80 (b) 20 (c) 10 (d) 50
122. The mean of 5 observations is 4.4 and their variance is 8.24. If three of the observations are 1, 2 and 6,
find the other two observations
(a) 4, 9 (b) 9, 4 (c) 4, 4 (d) 9, 9
123. If each of the observations x1 , x2 ,.....xn is increased by ' a ' , where a is a negative or positive number,
then the variance
(a) same (b) increased by a 2 times (c) decreased by a 2 times (d) None of these
124. The scores of batsman A in 10 different test matches were 38, 70, 48, 34, 42 55, 63, 46, 54, 44. Find
the standard deviation
(a) 10.61 (b) 8.6 (c) 9.61 (d) 6.8
Direction (125 - 126) : The mean and standard deviation of 100 observations were calculated as 40 and 5.1 ,
respectively by a student who took by mistake 50 instead of 40 for one observation
125. Find the correct mean.
(a) 38.9 (b) 37.9 (c) 39.9 (d) 36.9
126. Find the correct standard deviation.
(a) 4 (b) 6 (c) 3 (d) 5
127. Find the variance of the following data
6, 8, 10, 12, 14, 16, 18, 20, 22, 24
(a) 33 (b) 30 (c) 32 (c) 35
128. A frequency distribution table is given here
xi 4 8 11 17 20 24 32
fi 3 5 9 5 4 3 1
The variance = .........
(a) 45.8 (b) 46.8 (c) 47.8 (d) 48.8
Direction (129 - 131) : The frequency distribution table is given here
Class 30-40 40-50 50-60 60-70 70-80 80-90 90-100
Frequency 3 7 12 15 8 3 2
129. Find the mean
(a) 62 (b) 64 (c) 65 (d) 63
130. Find the variance.
(a) 205 (b) 203 (c) 201 (d) 204
131. Find the standard deviation
(a) 13.18 (b) 16.18 (c) 12.18 (d) 14.18
132. The standard deviation of a distribution is 30 and each item is raised by 3, then new S.D. is
(a) 32 (b) 28 (c) 27 (d) None of these

th
Office.: 606 , 6 Floor, Hariom Tower, Circular Road, Ranchi-1, Ph.: 0651-2562523, 9835508812, 8507613968
( BY R. K. MALIK’S NEWTON CLASSES
) 11
99
133. If the variance of 1, 2, 3, 4, 5, …, 10 is , then the standard deviation of 3, 6, 9, 12, …, 30 is
12
297 3 3 99
(a) (b) 33 (c) 99 (d)
4 2 2 12
134. If the standard deviation of 3, 8, 6, 10, 12, 9, 11, 10, 12, 7 is 2.71. Then the standard deviation of 30, 80, 60,
100, 120, 90, 110, 110, 120, 70, is
(a) 2.71 (b) 27.1 (c) ( 2.71) 10 (d) ( 2.71) 2
135. Find variance of the following data
Class interval 4-8 8-12 12-16 16-20
Frequency 3 6 4 7
(a) 19 (b) 20 (c) 21 (d) 23
Direction (136-138) : the frequency distribution table is given here.
Classes 1-10 11-20 21-30 31-40 41-50 51-60
Frequency 11 29 18 4 5 3
136. Find the mean
(a) 22.5 (b) 21.5 (c) 23.5 (d) 24.5
137. Find the variance
(a) 163.53 (b) 164.25 (c) 162.21 (d) 161.14
138. Find standard deviation
(a) 12.7 (b) 13.7 (c) 14.7 (d) 11.7
Directions (139-140) : The frequency distribution table is given here
xi 140 145 150 155 160 165 170 175

fi 4 6 15 30 36 24 8 2
139. Find the variance
(a) 51.7336 (b) 52.7136 (c) 50.7336 (d) 53.7236
140. Find the S.D.
(a) 7.26 (b) 7.36 (c) 7.16 (d) 7.56
Direction (141-142) : From a frequency distribution consisting of 18 observations, the mean and the standard
deviation were found to be 7 and4 respectively. But on comparison with the original data, it was found that a
figure 12 was miscopied as 21 in calculations.
141. Find the correct mean
(a) 9.5 (b) 8.5 (c) 6.5 (d) 7.5
142. Find the correct standard deviation
(a) 3.5 (b) 4.5 (c) 5.5 (d) 2.5
49
143. The variance of the numbers 2, 3, 11 and x is . Find the value of x
4
14 14 16 13
(a) 6, (b) 6, (c) 6, (d) 4,
3 5 3 5
144. The following values are calculated in respect of heights and weights of the students of a section of Class XI.
Height Weight
Mean 162.6 cm 52.36 kg
Variance 127.69 cm 2 23.1361 kg 2
Can we say that the weights show greater variation than the heights?
(a) Yes (b) No (c) Can’t say anything (d) None of these
145. Two plants A and B of a factory show following results about the number of workers and the wages
paid to them

th
Office.: 606 , 6 Floor, Hariom Tower, Circular Road, Ranchi-1, Ph.: 0651-2562523, 9835508812, 8507613968
12 ( )BY R. K. MALIK’S NEWTON CLASSES
A B
No. of workers 5000 6000
Average monthly wages Rs. 2500 Rs. 2500
Variance of distribution of wages 81 100
In which plant, A or B is there grater variability in individual wages?
(a) B (b) A (c) A = B (d) None of these
146. Coefficient of variation of two distributions are 60 and 70, and their standard deviations are 21 and 16,
respectively. What are their arithmetic means?
(a) 35, 22.85 (b) 35, 27.85 (c) 37, 22.85 (d) 37, 27.85
147. The mean and standard deviation of marks obtained by 50 students of a class in three subjects,
Mathematics, Physics and Chemistry are given below.
Subject Mathematics Physics Chemistry
Mean 42 32 40.9
Standard deviation 12 15 20
Which of these three subjects shows the highest variability in marks and which shows the lowest?
(a) Chemistry, Mathematics (b) Mathematics, Physics
(c) Chemistry, Physics (d) None of these
148. Find the C.V. of the following data
Size(in m) 10-15 15-20 20-25 25-30 30-35 35-40
No. of items 2 8 20 35 20 15
(a) 20.24 (b) 21.89 (c) 23.10 (d) 19.20
149. The mean height of 25 male workers in a factory is 61 cms, and the mean height of 35 female workers
in the same factory is 58 cms. The combined mean height of 60 workers in the factory is
(a) 59.25 (b) 59.5 (c) 59.75 (d) 58.75
150. The average salary of male employees in a firm was Rs.520 and that of females was 420. The mean
salary of all the employees was Rs.500.The percentage of male employees is
(a) 80 (b) 60 (c) 40 (d) 20.
151. The mean weight of 9 items is 15. If one more item is added to the data, the mean becomes 16. the
value of 10th item is
(a)35 (b) 30 (c) 25 (d) 20.
152. The value of median for the data
Income (in Rs.): 1000 1100 1200 1300 1400 1500
No. of persons : 14 26 21 18 28 14
(a) 1300 (b) 1200 (c) 1250 (d) 1150
153. The median from the following data is
Mid – Value Frequency Mid – value Frequency
115 6 165 60
125 25 175 38
135 48 185 22
145 72 195 3
155 116

(a) 153.79 (b) 153.91 (c) 165.18 (d) 165.93.

th
Office.: 606 , 6 Floor, Hariom Tower, Circular Road, Ranchi-1, Ph.: 0651-2562523, 9835508812, 8507613968
( )BY R. K. MALIK’S NEWTON CLASSES 13
154. The median from the following data is
Wages/week No. of workers Wages/week No. of workers
50 – 59 15 90 – 99 45
60 – 69 40 100 – 109 40
70 – 79 50 110 – 119 15
80 – 89 60
(a) 83.17 (b) 84.08 (c) 82.17 (d) 85.67
155. If the s.d. of a set of observations is 4 and if each observation is divided by 4, the s.d. of the new set of
observations will be
(a) 4 (b) 3 (c) 2 (d) 1
156. If s.d. of a set of observation is 8 and if each observation is divided by – 2, the s.d. of the new set of
observations will be
(a) – 4 (b) – 8 (c) 8 (d) 4
157. The variance of n observations x1 , x2 ,........xn is given by
2
1 n 1 n 1 n 1 n   1 n
(a) ∑ ( xi − x ) (b) ∑ xi2 − x −2 (c)  ∑ xi2 −  ∑ xi  ∑(x − x )
2 2
 (d) 2 i
n i =1 n i =1 n  i =1 n  i =1   n i =1

158. The geometric G of the numbers 7, 7 2 , 73 ,......, 7 n is


(a) 7( ) / 2
n+1
(b) greater than 7 n / 2 (c) less than 7 n / 2 (d) none of these
159. The semi-interquartile range of the data 3, 6, 5, 4, 2, 1, 7 is
(a) 4 (b) 3 (c) 2 (d) 1
160. The upper quartile for the following distribution
Size of items 1 2 3 4 5 6 7
Frequency 2 4 5 8 7 3 2
is given by the size of
th th th th
 31 + 1    28 + 1     31 + 1    28 + 1 
(a)   item (b) 3    item (c) 3    item (d)   item
 4    4    4   4 
161. The range and coefficient of range for the following data are respectively
Wages in thousands No. of workers
10 – 20 53
20 – 30 35
30 – 40 20
40 – 50 12
(a) 40 and 0.5 (b) 30 and 0.2 (c) 40 and 0.6 (d) 40 and 0.2
162. Quartile deviation for a frequency distribution is given by
1 1 1
(a) ( Q2 − Q1 ) (b) ( Q3 − Q1 ) (c) ( Q3 − Q1 ) (d) ( Q3 − Q1 )
4 3 2
163. The range and the coefficient of range for 6, 10, 15 25, 30, 32, 40, 46 respectively are
(a) 40, 0.77 (b) 40, 0.97 (b) 36, 0.77 (d) 36, 0.97
164. If the range of 15, 14, x, 25, 30, is 23, then the least possible value of x is
(a) 14 (b) 12 (c) 13 (d) 11
165. If the mean of the lower and upper quartile is 10 and semi-interquartile range is 5, then the value of
lower quartile and upper quartile are respectively
(a) 2 and 12 (b) 3 and 13 (c) 4 and 14 (d) 5 and 15

th
Office.: 606 , 6 Floor, Hariom Tower, Circular Road, Ranchi-1, Ph.: 0651-2562523, 9835508812, 8507613968
14 BY R. K. MALIK’S NEWTON CLASSES
( )
166. The range and the coefficient of range for 90, 50, 72, 69, 85, 100, 73, 85, 93 is
(a) 43, 0.43 (b) 50, 0.43 (c) 50, 0.33 (d) 43, 0.33
167. If the quartile deviation of a set of observations is 10 and the third quartile is 35, then the first quartile
is
(a) 24 (b) 30 (c) 17 (d) 15
168. The mean deviation of the data 3, 10, 10, 4, 7, 10, 5, from the mean is
(a) 2 (b) 2.57 (c) 3 (d) 3.75
169. The mean deviation about median from the data 340, 150, 210, 240, 300, 310, 320, is
(a) 50 (b) 52.8 (c) 55.4 (d) 45
170. For a frequency distribution, the mean deviation about mean is computed by

(a)
∑d i
(b)
∑fd i i
(c)
∑f d i i
(d)
∑f i

∑f i ∑f i ∑f i ∑f d i i

171. The mean deviation from the median is


(a) grater than measured from any other value (b) less than that measured from any other value
(c) equal to that measured from any other value (d) maximum if all observations are positive
172. The S.D. of 1, 2, 3, 4, 5, 6, is
35 35
(a) (b) 3 (c) 3 (d)
12 12
173. If each observation of a raw data whose variance σ 2 is multiplied h, then the variance of the new set is
(a) σ 2 (b) h 2σ 2 (c) hσ 2 (d) h + σ 2
174. If S.D. of x1 , x2 ,...xn is 5, then the S.D. of x1 + 5, x2 + 5, x3 + 5,..... xn + 5, is
(a) 0 (b) 10 (c) 5 (d) 25
175. For a frequency distribution, standard deviation is computed by applying the formula

∑ f (x − x)
2
∑ f (x − x) ∑ f (x − x) ∑ f (x − x)
2
i i i i i i i i
(a) (b) (c) (d)
∑f i ∑f i ∑f i ∑f i

176. If the S.D. of a set of observation is 8 and if each observation is divided by – 2, then S.D. of the new
set of observation will be
(a) – 4 (b) – 8 (c) 8 (d) 4
177. If Q.D. is 16, then the most likely value of S.D. will be
(a) 24 (b) 42 (c) 10 (d) 48
178. If the standard deviation of 1, 2, 3, 4,….10, is σ , then the standard deviation of 11, 12 , 13, 14,…..,20
is
(a) σ + 10 (b) 10σ (c) σ (d) σ 2
179. The CV for the set of observation 55, 54, 52, 53, 56, 58, 52, 51, 49, is
(a) 2.64 % (b) 3.74 % (c) 4.98 % (d) 5.78 %
180. In a set of observations, S.D. = 7 and mean is 28, then coefficient of variation is
1
(a) 4 % (b) % (c) 25 % (d) 12.5 %
4
181. In a set of observation coefficient of variation is 16 and mean is 25, then the variance is
(a) 4 (b) 8 (c) 12 (d) 16
182. For a given distribution of marks, mean is 35.16 and its standard deviation is 19.76. Then coefficient
of variation is

th
Office.: 606 , 6 Floor, Hariom Tower, Circular Road, Ranchi-1, Ph.: 0651-2562523, 9835508812, 8507613968
( BY R. K. MALIK’S NEWTON CLASSES
) 15
35.16 19.76 35.16 19.76
(a) % (b) % (c) × 100 % (d) × 100 %
19.76 35.16 19.76 35.16
183. If the CV and standard deviation of a distribution are 50 and 20 respectively, then its mean is
(a) 40 (b) 30 (c) 20 (d) 35
184. Which of the following is a measure of dispersion?
(a) Mean (b) Median (c) Mode (d) Standard deviation
185. The median of a set of distinct observations is 20.5. If each of the largest 4 observation of the set is
increased by 2, then the median of the new set
(a) Is increased by 2 (b) Is decreased by 2
(c) Is two times the original median (d) Remains the same as that of the original set
186. The Q.D. of the data 3, 5, 9, 3,21, 19, 7, 4, 18, 12, 10 is
(a) 7 (b) 9 (c) 7.5 (d) 9.5
1
187. The highest score of a certain data exceeds the lowest score by 16 and coefficient of range is . The
3
sum of he highest score and the lowest score is
(a) 36 (b) 48 (c) 24 (d) 18
188. The quartile deviation of daily wages (in Rs.) of 7 persons given 12, 7, 15, 10, 17, 17, 25, is
(a) 3.5 (b) 9 (c) 5 (d) 14.5
189. The coefficient of range for following data is
Age-group No. of persons
35-40 40
40-45 82
45-50 112
50-55 74
55-60 63
(a) 0.35 (b) 0.18 (c) 0.29 (d) 0.26
190. If 25% of the items are less than 20 and 25% are more than 40, the quartile deviation is
(a) 20 (b) 30 (c) 40 (d) 10
191. The mean deviation of the following data from mean is
CI : 0 – 5 5 – 10 10 – 15 15 – 20 20 – 25
fi : 3 4 8 10 5
(a) 5 (b) 4 (c) 6 (d) 3
192. Mean deviation (approximately) about mean from the following data
xi 3 9 17 23 27
fi 8 10 12 9 5
(a) 6 (b) 7 (c) 8 (d) 9
193. Following are the marks obtained by 9 students in Mathematics test: 50, 69, 20, 33, 53, 39, 40, 65, 59,
then mean deviation from the median is
(a) 9 (b) 10.5 (c) 12.67 (d) 14.76
194. The mean deviation about mean of the data
xi 5 10 15 20 25
fi 8 7 8 11 6

(a) 4.15 (b) 3.80 (c) 5.75 (d) 7.95


th
Office.: 606 , 6 Floor, Hariom Tower, Circular Road, Ranchi-1, Ph.: 0651-2562523, 9835508812, 8507613968
16 ( BY R. K. MALIK’S NEWTON CLASSES
)
195. If µ is the mean of a distribution, then ∑ f i ( yi − µ ) is equal to
(a) M.D. (b) S.D. (c) 0 (d) none of these
196. The mean, median and standard deviation of a distribution are 16, 20 and 4 respectively. If every value
is increased by 2, then
(a) mean, median and S.D. each will be increased by 2.
(b) mean and median will be increased by 2 but S.D. will remain same.
(c) mean will be increased by 2 where as median and S.D. will be same.
(d) all the three will remain same.
197. If standard deviation of a variate x is 10, then S.D. of the variate (50 + 50x) will be
(a) 10 (b) 50 (c) 500 (d) 100
198. The mean of 5 observation is 4.4 and their variance is 8.24. If three observation are 1, 2 and 6, the
other two observation are
(a) 4 and 8 (b) 4 and 9 (c) 5 and 7 (d) 5 and 9
199. If the standard deviation of 0, 1, 2, 3,…..,9 is K, then the standard deviation of 10, 11, 12, 13 ….19 is
(a) K (b) K + 10 (c) K + 10 (d) 10K
200. If M.D. is 12, the approximate value of S.D. will be
(a) 15 (b) 12 (c) 24 (d) 42
201. The variance of α , β and γ is 9, then variance of 5α , 5β and 5γ is
9 5
(a) 45 (b) (c) (d) 225
5 9
202. The variance of the following frequency distribution is
CI 0 – 6 6 – 12 12 – 18
fi 2 4 6
(a) 24 (b) 12 (c) 20 (d) 25
203. The mean of 100 observation is 50 and their standard deviation is 5. The sum of the squares of all
observation is
(a) 50000 (b) 250000 (c) 252500 (d) 255000
[TRIUMPH MATHEMAICS Mr. Vinod Singh & Shweta Pawar]
204. If the variance of the data 2, 4, 6, 8, 17 is 23.33, then the variance of 4, 8, 12, 16, 34, is
(a) 23.3 (b) 25.33 (c) 46.66 (d) 93.32
205. For two data sets each of size 5, the variance are given to be 4 and 5 and the corresponding means are
given to be 2 and 4 respectively. The variance of the combined data set is
13 5 11
(a) 6 (b) (c) (d)
2 2 2
206. The standard deviation of the observation 22, 26, 28, 20, 24, 30 is
(a) 2 (b) 2.4 (c) 3 (d) 3.42
207. Suppose a population A has 100 observation 101, 102,….,200 and another population B has 100
observation 151, 152, …..,250. If VA and VB represent the variance of two population respectively,
V
then A is
VB
9 4 2
(a) 1 (b) (c) (d)
4 9 3
208. The mean and S.D. of the marks of 200 candidates were found to be 40 and 15 respectively. Later, it
was discovered that a score of 40 was wrongly read as 50. The correct mean and S.D. respectively are
(a) 14.98, 39.95 (b) 39.95, 14.98 (c) 39.95, 224.5 (d) None of these
th
Office.: 606 , 6 Floor, Hariom Tower, Circular Road, Ranchi-1, Ph.: 0651-2562523, 9835508812, 8507613968
( ) BY R. K. MALIK’S NEWTON CLASSES 17
209. Coefficients of variance of two distributions are 50 and 60 and their arithmetic means are 30 and 25
respectively. Difference of their standard deviations is
(a) 0 (b) 1 (c)1.5 (d) 2.5
210. The quartile deviation for the following data is
x 2 3 4 5 6
f 3 4 8 4 1
1 1
(a) 0 (b) (c) (d) 1
4 2
211. The quartile deviation of daily wages (in Rs.) of 7 persons given 12, 7, 15, 10, 17, 19, 25 is
(a) 14.5 (b) 5 (c) 9 (d) 4.5
212. A batsman scores 38, 70, 48, 34, 42, 55, 63, 46, 54, 44, runs in 10 innings, then the mean deviation
from median is
(a) 8.6 (b) 6.4 (c) 10.6 (d) 9.6
213. For a frequency distribution, standard deviation is computed by applying the formula
2
 ∑ fd  ∑ fd 2 ∑ fd 2
 ∑ fd 2 
(a) σ =  − (b) σ = −
 ∑ f  ∑ f ∑f  ∑ f 
   
2 2
 ∑ fd  ∑ fd 2 ∑ fd 2
 ∑ fd 
(c) σ =  − (d) σ = −
 ∑ f  ∑f ∑f  ∑ f 
   
214. The standard deviation of 25 numbers is 40. If each of the numbers is increased by 5, then the new
standard deviation will be
21
(a) 40 (b) 45 (c) 40 + (d) None of these
25
215. What is the cumulative frequency curve of statistical data commonly called?
(a) Cartogram (b) Histogram (c) Pictogram (d) Ogive
216. What is the mode for the data 20, 20, 20, 21, 21, 21, 21, 21, 22, 22, 22, 22, 22, 22, 22, 23, 23, 23, 23,
23, 24, 24, 25?
(a) 7 (b) 21 (c) 22 (d) 25
217. The frequency distribution of given numbers is
Value Frequency
1 5
2 4
3 6
4 f
If the mean is known to be 3, then the value of f is
(a) 3 (b) 7 (c) 10 (d) 14
218. The average of the squares of the numbers 0, 1, 2, 3, 4, ….n is
1 1 1 1
(a) n ( n + 1) (b) n ( 2n + 1) (c) ( n + 1)( 2n + 1) (d) n ( n + 1)
2 6 6 6
219. What is the geometric mean of 10, 40 and 60?
(a) 10 3 3 (b) 20 3 3 (c) 40 3 3 (d) 70 3 3
220. What is the median of the distribution 3, 7, 6, 9, 5, 4 and 2?
(a) 5 (b) 6 (c) 7 (d) 8
221. The S.D. of 15 items is 6 and if each item is decreased by 1, then standard deviation will be

th
Office.: 606 , 6 Floor, Hariom Tower, Circular Road, Ranchi-1, Ph.: 0651-2562523, 9835508812, 8507613968
18 ( )BY R. K. MALIK’S NEWTON CLASSES
91
(a) 5 (b) 7 (c) (d) 6
15
222. A variate X takes values 2, 9, 3, 7, 5, 4, 3, 2, 10 What is the median?
(a) 2 (b) 4 (c) 7 (d) 9
223. What is the geometric mean of the data 4, 8, 16 and 32?
(a) 2 (b) 4 (c) 8 (d) 16
224. What is the mean deviation from mean of the data 2, 9, 9, 3, 6, 9 and 4?
(a) 2.23 (b) 2.57 (c) 3.23 (d) 3.57
225. The adjoining diagram gives a pie chart representing the units of electricity sold to different classes in
a month. The angle of the sector corresponding to supply for industries is

(a) 25° (b) 46°


(c) 60° (d) 90°

226. What is the arithmetic mean of first 16 natural numbers with weights being the number itself?
(a) 17/2 (b) 33/2 (c) 11 (d) 187/2
227. In a batch of 15 students, If the marks of 10 students, who passed are 70, 50, 95, 40 60, 70, 80, 90, 75,
80, then the median marks of the all 15 students is
(a) 40 (b) 50 (c) 60 (d) 70
228. The distributions X and Y with total number of observation 36, 64 and mean 4, 3, respectively are
combined. What is the mean of the resulting distribution of all the 100 observations?
(a) 3.26 (b) 3.32 (c) 3.36 (d) 3.42
229. The geometric mean of three numbers was computed as 6. It was subsequently found that in this
computation, a number 8 was wrongly read as 12. What is the correct geometric mean?
(a) 4 (b) 3 5 (c) 2 3 18 (d) None of these
230. The harmonic mean of two numbers is 21.6. If one of the numbers is 27, what is the other number?
(a) 16.2 (b) 17.3 (c) 18 (d) 20
231. What is the standard deviation of numbers 7, 9, 11, 13 and 15?
(a) 2.2 (b) 2.4 (c) 2.6 (d) 2.8
232. If the three observations are 3, −6 and −6, then what is their harmonic mean?
(a) 0 (b) −1/ 2 (c)3 (d) None of these
233. The average age of 20 students in a class is 15 yr. If the teacher’s age is included, the average incrases
by one. What is the teacher’s age?
(a) 30 yr (b) 21 yr (c) 42 yr (d) 36 yr
234. The data below records the itemwise quarterly expenditure of a private organization.
Items of expenditure Amount (in Rs. lakh)
1. Salaries 6
2. TA & DA 4.9
3. House rent and postage 3.6
4. All other expenses 5.5
Total 20
The data is represented by a pie diagram. What is the sectorial angle of the sector with largest area?
(a) 120° (b) 108° (c) 100° (d) 90°

th
Office.: 606 , 6 Floor, Hariom Tower, Circular Road, Ranchi-1, Ph.: 0651-2562523, 9835508812, 8507613968
BY R. K. MALIK’S NEWTON CLASSES
( ) 19
235. The mean of a set of observations is x . If each observation is divided by α ≠ 0 and then is increased
by 10, then the mean of the new set of observations is
x x + 10 x + 10α
(a) (b) (c) (d) α x + 10
α α α
236. The arithmetic mean of data with observations a, a + d , a + 2d ,......, a + 2md is
1 1
(a) a + md (b) a + ( m − 1) d (c) a + md (d) a + ( m + 1) d
2 2
237. Let x be the mean of n observations x1 , x2 ,.....xn . If ( a − b ) is added to each observations, what is the
mean of new set of observations?
(a) 0 (b) x (c) x − ( a − b ) (d) x + ( a − b )
238. If the median of x / 5, x, x / 4, x / 2, and x / 3, where, x > 0, is 8, then the value of x is equal to
(a) 26 (b) 24 (c) 20 (d) 32
239. Following table gives the mean and variance of monthly demand for four products A,B,C and D in a
supermarket
Product A B C D
Mean demand 60 90 80 120
Variance 12 25 36 16
For which product the demand in consistent?
(a) Product A (b) Product B (c) Product C (d) Product D
240. The mean of 7 observations is 10 and that of 3 observations is 5. What is the mean of all the 10
observations?
(a) 15 (b) 10 (c) 8.5 (d) 7.5
241. Geometric mean of 3,9 and 27 is :
(a) 18 (b) 6 (c) 9 (d) None of these
242. Harmonic mean of 2, 4 and 5 is :
(a) 421 (b) 3.16 (c) 2.98 (d) None of these
243. The quartile deviation for the data :
x 2 3 4 5 6
f 3 4 8 4 1
1 1
(a)0 (b) (c) (d) 1
4 2
True/ False
244. The mode of a distribution is the value that has the greatest concentration of frequencies.
245. Median is not affected by extreme values.
246. Standard deviation can be calculated from any average.
247. If both regression coefficients are positive then their arithmetic mean cannot be less than the
correlation coefficient.
Column Matching
248. Arithmetic mean and standard deviation of x1 , x2 ,...........xn are 15 and 3 respectively. For
3 − 2 x1 ,3 − 2 x2 ,......., 3 − 2 xn ,
(i) A.M. (a) 6
(ii) S.D. (b) −27
(c) 9
(d) 8

th
Office.: 606 , 6 Floor, Hariom Tower, Circular Road, Ranchi-1, Ph.: 0651-2562523, 9835508812, 8507613968
JEE (MAIN & ADV.), MEDICAL + BOARD, NDA, X & IX
Enjoys unparalleled reputation for best results
in terms of percentage selection
www.newtonclasses.net

SOLUTION OF STATISTICS ( )
Only one option is correct.
1. Ans. (c) Out of the given alternatives, mean – deviation is not a measure of central tendency.
2. Ans. (b) The class mark of a class in any distribution is the value exactly at the middle of the class
Interval. It is also called the mid – value to the class interval and is equal to the average of the lower
limit and the upper limit of that class – interval. Therefore, the average of any two consecutive class
marks gives the upper limit of the first class – interval and the lower limit of the next class interval.
6 + 10
∴ For the given class marks, we have : The lower limit of 2nd class interval = = 8 ; The upper
2
10 + 14
limit of 2nd class interval = = 12 ; And so, the class size of the 2nd class interval = upper limit −
2
10 + 14
lower limit = 12 − 8 = 4 . The lower limit of 3rd class interval = = 12 ; The upper limit of 3rd class
2
14 + 18
interval = = 16 ; And so, the class size of the 3rd class interval = upper limit − lower
2
limit = 16 − 12 = 4 . Similarly, the class size of each class interval = 4 .
3. Ans. (a) Number of observations, n = 10 . Mean of these 10 observations is x = 16.3 . ∴Sum of all the
10 observations = nx = 10 × 16.3 = 163 . Corrected sum = 163 − 32 + 23 = 154 . ∴Corrected Mean,
154
x′ = = 15.4 .
10
n

2 n−1
∑x i
1 + 3 + 32 + .... + 3n −1 1( 3 − 1)
n

4. Ans. (a) A.M. of 1,3, 3 ,.....3 is given by x = i =1


= = .
n n ( 3 − 1) n
[ ∴ S n = 1 + 3 + 32 + .... + 3n −1 = Sum to n terms of a G.P. with first term a = 1 and common ratio r = 3 ]
3n − 1
= .
2n

5. Ans. (b) We have : Mean x =


∑ f x = (5×2) +(8×5) +(10×8) +(12×22) +( x ×7) +( 20×4) +( 25×2)
i i 1

∑f i 2 +5+8+ 22 +7 + 4 + 2
524 + 7 x1
⇒ 12.58 = ⇒ 7 x1 = 105 ⇒ x1 = 15
50
6. Ans. (c) There are 100 observations namely 1, 2, 3, ….., 98, 99, x . The average of these 100
1 + 2 + 3 + .... + 98 + 99 + x
observations is given by x = ⇒ 100 × 100x = (1 + 2 + 3 + .... + 98 + 99 ) + x
100
99 × 100 4950 50
[∵ x = 100 x ( given) ] ⇒ 9999 x = 1 + 2 + 3 + .... + 98 + 99 = = 4950 ⇒ x = =
2 9999 101

7. Ans. (b) We have : Mean x =


∑f x i i
=
( 3 × 6 ) + ( 5 × 8 ) + ( 7 ×15 ) + ( 9 × f1 ) + (11× 8 ) + (13 × 4 )
∑f i 6 + 8 + 15 + f1 + 8 + 4
303 + 9 f1
⇒ 7.68 = ⇒ 314.88 + 7.68 f1 = 303 + 9 f1 ⇒ 11.88 = 1.32 f1 ⇒ f1 = 9 .
41 + fi
8. Ans. (a) Mean of 27 + x, 31 + x,89 + x , 107 + x, 156 + x is 82
th
Office.: 606 , 6 Floor, Hariom Tower, Circular Road, Ranchi-1, Ph.: 0651-2562523, 9835508812, 8507613968
2 ( BY R. K. MALIK’S NEWTON CLASSES
)


( 27 + x ) + ( 31 + x ) + ( 89 + x ) + (107 + x ) + (156 + x ) = 82 ⇒ 410 + 5 x = 410 ⇒ 5 x = 0 ⇒ x = 0 .
5
Now, mean of 130 + x,126 + x, 68 + x, 50 + x,1 + x = mean of 130, 126, 68, 50, 1 [∵ x = 0]
130 + 126 + 68 + 50 + 1 375
= = = 75
5 5
9. Ans. (b) The number σ 2 which is means of the squares of the deviations from mean is called the
variance and is denoted by
10. Ans. (b) The A.M. of the 9 terms = 15 . ∴ Sum of all the 9 terms = 15 × 9 = 135 . Let the added term be
135 + A 135 + A
A . Then, the sum of all the 10 terms = 135 + A . ∴ Of all t he 10 terms, x = ⇒ 16 =
10 10
[∵ x = 16 ( given) ] A = 160 − 135 = 25
100

∑ xi ∑x i
11. Ans. (b) Let x be the mean of the 100 items x1 , x2 ,....., x100 . Then, n = 100 . Now, x = i =1
= i =1

n 100
100

∑x i 100
⇒ 49 = i =1
[∵ x = 49 ( given) ] ⇒ ∑ xi = 4900 ….(i)
100 i =1
100 100 100
Now, corrected value of ∑x
i =1
i is given by ∑ x′ = ∑ x − ( 38 + 22 + 50 ) + ( 60 + 70 + 80 )
i =1
i
i =1
i

100

5000 ∑x 1
i
= 4900 − 110 + 210 = 5000 . ∴ Correct Mean x′ = = 50 i =1
=
100 100
12. Ans. (c) We draw the following table for the given Continuous Frequency Distribution :
Class Midvalue of Class Frequency ( f i ) f i xi
Interval ( xi )
0 – 20 10 17 170
20 – 40 30 fi 30 f1
40 – 60 50 32 1600
60 – 80 70 f2 70 f 2
80 – 100 90 19 1710
∑f i = 68 + f1 + f 2 ∑fx i i = 3480 + 30 f1 + 70 f 2
Now, we have : ∑f i = 120 ( given) . ∴ 68 + f1 + f 2 = 120 ⇒ f1 + f 2 = 52 ….(i) Also, x = 50 (given)


∑fx i i
= 50 ⇒
3480 + 30 f1 + 70 f 2 
= 50 ∵ x =
∑ f x  ⇒ 3 f
i i
+ 7 f 2 = 252 ….(ii).
∑f i 120  ∑ f  i
1

Solving (i) and (ii), we get : f1 = 28, f 2 = 24 .


Clearly, the greater of the two frequencies f1 and f 2 is f1 whose value is 28.
13. Ans. (c) The A.M. of 7 consecutive integers starting with a i.e., of the numbers
a + ( a + 1) + ( a + 2 ) + .... + ( a + 6 ) 7 a + (1 + 2 + .... + 6 )
a, a + 1, a + 2,....., a + 6 is given by x1 = =
7 7
1
7a + × 6 × 7 1
2  
⇒m=
7
= a +3 ∴ 1 + 2 + .... + 6 = 2 × 6 × 7  ⇒ a = m − 3 ….(i) The A.M. of 11

th
Office.: 606 , 6 Floor, Hariom Tower, Circular Road, Ranchi-1, Ph.: 0651-2562523, 9835508812, 8507613968
(BY R. K. MALIK’S NEWTON CLASSES
) 3
consecutive integers starting with a + 2 i.e., of the numbers a + 2, a + 3,....., a + 12 is given by
( a + 2 ) + ( a + 3) + .... + ( a + 12 ) 11a + ( 2 + 3 + .... + 12 ) 1
x2 =
11
=
11
=a+
11
{(1 + 2 + 3 + .... + 12 ) − 1}
1 12 × 13  1 154
= ( m − 3) +  − 1 = ( m − 3 ) + × = ( m − 3) + 7 = m + 4
11  2  11 2
14. Ans. (b) The sum of the squares of the deviations is least when taken about the mean.
15. Ans. (d) On dividing each entry in a data by a non – zero number ' a ' the arithmetic mean of the new
data is divided by a .
16. Ans. (b) We know that : If each observations is increased by a constant a , then the mean is also
increased by a . ∴ If the mean of 12 numbers = 26 , then the mean when each number is increased by
5 = 26 + 5 = 31 .
17. Ans. (b) The mean of four observations is x = 20 . The mean of these four observations, when C is
added to each of these observations = 22 . Now, if each observations is increased by C , then the mean
is also increased by C . ∴ x + C = 22 ⇒ C = 22 − x = 22 − 20 = 2 .
18. Ans. (b) Number of student in the class, n = 10 . Mean marks of the class, x = 66 . ∴ Total marks ( of
all the 10 students) = n x = 10 × 66 = 660 . Total marks of all the students of new class
= 660 − 34 − 37 + 52 = 641 . Number of students in the new class formed, n′ = 9 . ∴ Mean marks of the
641
new class formed, x′ = = 71.22 .
9
19. Ans. (d) The average weight of 35 students = 40 kg. ∴ Total weight of all the 35 students = ( 35 × 40 )
kg = 1400 kg. Let W kg be the weight of the teacher. The average weight of 35 students +1 teacher
 1
=  40 +  kg = 40.5 kg. ∴ Total weight of all the 36 persons ( which includes 35 students and 1
 2
teacher ) = ( 36 × 40.5 ) kg = 1458 kg ⇒ 1400 + W = 1458 ⇒ W = 1458 − 1400 = 58 . ∴ The weight of the
teacher is 58 kg.
20. Ans. (c) Let the number of boys and girls in the class be n1 and n2 respectively. And let x1 and x2 be
the mean marks of the boys and the girls respectively. Then, n1 = 70, n1 + n2 = 100 and so
n2 = 100 − 70 = 30 . Also, x1 = 75 . The mean marks of all the 100 students of the class is X = 72 .

Now, X =
n1 x1 + n2 x2
⇒ 72 =
70 × 75 + 30 × x2
⇒ x2 =
( 70 ×100 ) − ( 70 × 75) = 65 . ∴ The mean marks
n1 + n2 100 30
of the girls in the class = 65 .
21. Ans. (d) The given set of 50 numbers can be divided into 2 groups. The first group consists of two
numbers 45 and 55. The number of items in this group, n1 = 2 . The A.M. of this group,
45+55
x1 = = 50 . The second group consists of remaining 48 numbers i.e., n2 = 48 . Let the A.M. of this
2
group be x2 . The number of items in the combined group = n1 + n2 = 50 . The combined A.M. X = 38
n1 x1 + n2 x2 2 × 50 + 48 × x2
( given). Now, X = ⇒ 38 = ⇒ x2 = 37.5 .
n1 + n2 50
22. Ans. (a) Let n1 and n2 respectively be the numbers of males and females employed in the factory.
The average salary of the males is x1 = Rs 210 ; The average salary of the females is x2 = Rs.190 ; The
n1 x1 + n2 x2
average salary of all the workers in the factory is X = Rs 206 . Now we know that X =
n1 + n2

th
Office.: 606 , 6 Floor, Hariom Tower, Circular Road, Ranchi-1, Ph.: 0651-2562523, 9835508812, 8507613968
4 ( BY R. K. MALIK’S NEWTON CLASSES
)
210n1 + 190n2 n 1
⇒ 206 = ⇒ 206 n1 + 206 n2 = 210 n1 + 190 n2 ⇒ 16 n2 = 4n1 ⇒ 2 = . ∴ Percentage of
n1 + n2 n1 4
 n2 
 n1   n 
females employed in the class =  = 100  % =  1 × 100  % [ Dividing Nr. & Dr. by n1 ]
 n1 + n2   1 + n2 
 n 
 1 
 1 
 4  1 
= × 100  % [ Using (i)] =  × 100  % = 20%
1 5 
 1+ 
 4 
23. Ans. (c) There are 7 observations in the given data ie., n = 7 , which is odd. Arranging the given
 n +1
ungrouped data in an ascending order, we obtain : 1, 3, 4, 5, 8, 9, 10. The median item =   th
 2 
 7 +1  th th
item =   th item = 4 item. Hence, median = 5 ( which is the 4 item in the arrayed sequence )
 2 
24. Ans. (c) There are 5 observations in the given data i.e., n = 5 , which is odd. Arranging the given
 n +1
ungrouped data in an ascending order, we obtain : 10, 18, 21, 35, 42. The median item =   th
 2 
 5 +1
item =   th item = 3rd item. = 21
 2 
25. Ans. (d) The given data has already been arranged as a sequence in an ascending order. Number of
n n 
observations n = 10 ( which is even). ∴ Median = A.M of th and  + 1 th items = A.M. of 5th and
2 2 
1  1  5  2x + 2
6th items [∵ n = 10] =  x −  +  x +   = = x + 1 . But Median = 23 ( given)∴ x + 1 = 23
2  2  2  2
⇒ x = 22 .
26. Ans. (d) The observation which occurs most frequently in a sequence is known as the mode of the
data.
27. Ans. (d) In the given ungrouped data, the item 7 is repeated maximum number of times. ∴ Mode of
the given series is 7.
28. Ans. (b) From the given ( Discrete Frequency Distribution) table it is clear that there is maximum
number of children (16) having weight 46 kg. i.e.,, the weight of 46 kg has the maximum frequency.
Hence, the mode of the given distribution = 46 .
29. Ans. (c) In the given data, we have : Modal class i.e.,the class with the maximum frequency is 15–20 ;
l = lower limit of the modal class = 15 ; f m = frequency of the modal class = 30 ; f m −1 = frequency of
the class preceding the modal class = 25 ; fm+1 = frequency of the class succeeding the modal class = 16

h = width of the modal class = 5 . ∴ Mode = l +


f m − f m −1
× h = 15 +
( 30 − 25 ) × 5
2 f m − f m −1 − f m +1 2 × 30 − 25 − 16
25
= 15 + = 15 + 1.316 ≈ 16.3 .
19
1
30. Ans. (d) The empirical relation between Mean, Median and mode is Median = ( Mode + 2 Mean)
3
1 192
Here, Mean = 66 , Mode = 60 . ∴ Median = ( 60 + 2 × 66 ) = = 64 .
3 3

th
Office.: 606 , 6 Floor, Hariom Tower, Circular Road, Ranchi-1, Ph.: 0651-2562523, 9835508812, 8507613968
BY R. K. MALIK’S NEWTON CLASSES
( ) 5
31. Ans. (c) The Mean, Median and Mode are related as Mode = 3 Median −2 Mean. Here, Mean = 21 ,
Median = 22 . ∴ Mode = 3 × 22 − 2 × 21 = 66 − 42 = 24 .
32. Ans. (c) For a moderately skewed distribution, the mean, median and mode are related as
Mode = 3 Median −2 Mean. Here, Median = 36 , Mean = 34 . ∴ Mode = 3 × 36 − 2 × 34 = 108 − 68 = 40
33. Ans. (a) The given set of observations is −1, 0 , 4 . Number of observations, n = 3 . Mean,
n

∑x i
−1 + 0 + 4
x= i =1

n
=
3
(
= 1 . The values of xi − x are −2, −1,3 . The values of xi − x are 2, 1, 3.)
n 3

∑ xi − x ∑ x −x i
2 +1+ 3
Now, M.D. x = () i =1

n
= i =1

3
=
3
= 2 . Thus, the mean deviation from the mean = 2
n

∑x i
34. Ans. (b) Mean of the given ungrouped data ( having 6 observations) is given by x = i =1

n
6 + 8 + 12 + 15 + 10 + 9
=
6
= 10 [∵ n = 6] . The values of xi − x are −4, −2, 2, 5, 0, −1 . ∴ The values of ( )
n 6

∑ xi − x ∑ x −x i
4 + 2 + 2 + 5 + 0 + 1 14
xi − x are 4, 2, 2, 5, 0,1. And so, M.D., x = () i =1

n
= i =1

6
=
6
=
6
≈ 2.33 .
n

∑x i
35. Ans. (b) The mean of the given ungrouped data ( having 10 observations ) is x = i =1

n
18 + 20 + 12 + 14 + 19 + 22 + 26 + 16 + 19 + 24 190
=
10
=
10
= 19 . [∵n = 10] . The values of xi − x ( )
are −1,1, −7, −5, 0,3, 7, −3, 0, 5 . ∴ The values of xi − x are 1,1, 7,5, 0,3, 7, ,3, 0,5 . And so,
n

∑ x −x i
1 + 1 + 7 + 5 + 0 + 3 + 7 + 3 + 0 + 5 32
M. D., x = ()n
i =1

10
==
10
= 3.2

36. Ans. (c) The given ungrouped data is 38, 70, 48, 34, 42, 55, 63, 46 , 54,44. The mean of this data (

having 10 observations) is given by x =


∑ x = ∑ x [∵n = 10]
i i

n 10
38 + 70 + 48 + 34 + 42 + 55 + 63 + 46 + 54 + 44 494
=
10
=
10
= 49.4 . The values of xi − x are : −11, 20.6 , ( )
−1.4, − 15.4, −7.4 , 5.6, 13.6, −3.4, 4.6, −5.4 . ∴ The values of xi − x are : 11.4, 20.6, 1.4, 15.4, 7.4,
10

∑ x −x i
11.4 + 20.6 +1.4 +15.4 + 7.4 + 5.6 +13.6 + 3.4 + 4.6 + 5.4
5.6, 13.6, 3.4, 4.6, 5.4 and so, M.D. x = () i =1

n
=
10
88.8
= = 8.88 i.e., the mean deviation is 8.88.
10
37. Ans. (c) We have : Standard deviation, σ = V ⇒ Variance, V = σ 2
38. Ans. (c) The given observations are 35, 40, 42, 36, 27. clearly, there are 5 observations i.e., n = 5

Mean, x =
∑x i
=
35 + 40 + 42 + 36 + 27 180
= = 36 . The values of xi − x are −1, 4, 6, 0, −9 . ∴ The ( )
n 5 5

th
Office.: 606 , 6 Floor, Hariom Tower, Circular Road, Ranchi-1, Ph.: 0651-2562523, 9835508812, 8507613968
6 ( ) BY R. K. MALIK’S NEWTON CLASSES
∑ ( x − x)
2
1 + 16 + 36 + 0 + 81
( )
2 i
values of xi − x are 1,16,36, 0,81. Now, Variance σ 2 = 26.8 = =
n n
39. Ans. (c) The given observations are 2, 4, 6, 8, 10. Clearly, there are 5 observations i.e,, n = 5 .

Mean, x =
∑x i
=
2 + 4 + 6 + 8 + 10 30
= = 6 . The values of ( x − x ) are −4, − 2, 0, 2, 4 . The values of
i
n 5 5
∑ ( x − x)
2
16 + 4 + 0 + 4 + 16 40
( x − x)
2 i
i are 16, 4, 0, 4, 16. Now, variance σ 2 = =8. = =
n 5 5
40. Ans. (d) The given observations are 7, 9, 11, 13, 15. Clearly, there are 5 observations i.e., n = 5 .

Mean, x =
∑x i
=
7 + 9 + 11 + 13 + 15 55
= (
= 11 . The values of xi − x are −4, −2, 0, 2, 4 . ∴ The values )
n 5 5

∑ ( x − x)
2
16 + 4 + 0 + 4 + 16 40
( x − x)
2 i
of i are 16, 4, 0, 4, 16. Now, variance, σ 2 = = = = 8 . And
n 5 5
so, standard deviation σ = 8 = 2 2 2 ×1.414 ≈ 2.8
41. Ans. (b) We know that : If in a series, each of the observations is increased by a constant k , then the
value of the standard deviation remains unchanged . So, when each number in the given set of 25
numbers is increased by 5, then the standard deviation remains unchanged i.e., the new standard
deviation will still be 40.

42. Ans. (c) We have : n = 4; ∑x = 4 + ( −3) + 6 + 1 = 8 ; x =


∑x i
=
8
= 2; ∑x 2 2
= 42 + ( −3) + 62 + 12
i 1
n 4
1/ 2
 ∑ xi2 2
  62 
1/ 2
 46 
1/ 2

= 16 + 9 + 36 + 1 = 62 . ∴ Standard deviation σ =  −x  =  − 22  =  = 11.5


 n  4   4 
43. Ans. (c) The variance of 20 observations is given as σ 2 = 10 . ∴ The S.D. of these 20 observations is
σ = 10 . And so, the S.D. of new set of observations ( obtained by multiplying each observations by
3) σ ′ = 3σ = 3 10 . [ ∵ if each observations in a series is multiplied by a constant α , then the S.D. is

( )
2 2
multiplied by α ] . ∴ The variance of new set of observations = (σ ′ ) = 3 10 = 90 .
44. Ans. (c) The S.D. of 1, 2, 3, 4, 5, 6, 7 is 2. ∴ The S.D. of
(11×1) , (11× 2 ) , (11× 3) , (11× 4 ) , (11× 5) , (11× 6 ) , (11× 7 ) is 11× 2 . [∵ if each observation in a series is
multiplied by a constant α , then the S.D. is multiplied by α ] i.e., the S.D. of 11, 22, 33, 44, 55, 66,
77 is 22. And so, the S.D. of (11 + 1) , ( 22 + 1) , ( 33 + 1) , ( 44 + 1) , ( 55 + 1) , ( 66 + 1) , ( 77 + 1) is 22. [∵ if a
constant k is added to each observation of a series, then the S.D. remains unchanged ]
i.e.., the S.D. of 12, 23, 34, 45, 56, 67, 78 is 22.
45. Ans. (b) Standard deviation of n observations is σ = 2 . ∴ Variance σ 2 = 4 . Now,
n n 2
 n   n

∑ x
2
∑ 2
i x  ∑ xi 
2
i  ∑x  i
100  20 
2
 n n

 ∑ i ∑ xi2 = 100 ( given ) 
2
σ = i =1
−x = i =1
−  i=1  ∵ x = i =1
 ⇒4= −  ∵ x = 20,
n n  n   n  n  n   i =1 i =1 
   
 
⇒ 4n 2 = 100n − 400 ⇒ n 2 − 25n + 100 = 0 ⇒ ( n − 5 )( n − 20 ) = 0 ⇒ n = 5 or 20.
−1 + 1 + 2 + a + b a+b+2
46. Ans. (a) Let the other two numbers be a and b . Now, mean x = ⇒0=
5 5

th
Office.: 606 , 6 Floor, Hariom Tower, Circular Road, Ranchi-1, Ph.: 0651-2562523, 9835508812, 8507613968
( ) BY R. K. MALIK’S NEWTON CLASSES 7
5

∑x 2
i
2 ( −1)
2
+ 12 + 2 2 + a 2 + b 2
∵Mean = 0 ( given )  ⇒ a + b = −2 ….(i) . Variance, σ = 2 i =1
−x = − 02
5 5
a2 + b2 + 6
⇒2= [∵ Variance, σ 2 = 2 ( given) ] ⇒ a 2 + b 2 = 4 …..(ii) Solving (i) and (ii) we get :
5
( a = −2, b = 0 ) or ( a = 0, b = −2 ) . ∴ The other two numbers are −2 and 0
47. Ans. (d) We know that : If each observation is multiplied by a non – zero real number α , then the
mean of the new observations is α x . ∴ If the mean of the given series of observations is 8, then the
mean of the sequence obtained by multiplying each observation by 3 is equal to 3 × 8 i.e., 24.
Also, if in a sequence, each of the observations is multiplied by a scalar α , then the value of the
2
standard deviation is multiplied by α and consequently the value of the variance is multiplied by α .
∴ If the variance of the given sequence of observations is 16,then the variance of the sequence
obtained by
multiplying each observation by 3 is equal to 32 ×16 i.e., 144.
48. Ans. (c) For the given continuous Frequency Distribution, we have : h = width of the class – interval
= 10 . Let the assumed mean be A = 25 ( Mid – point of the class having maximum frequency is
generally taken as Assumed Mean). We now prepare the following table :
Class Mid – Frequency xi − A ui2 f i ui f i ui2
value of u i =
fi h
Class
x − 25
Interval = i
10
0 – 10 5 1 −2 4 −2 4
10 – 20 15 3 −1 1 −3 3
20 – 30 25 = A 4 0 0 0 0
30 – 40 35 2 1 1 2 2
N = ∑ fi ∑ fu i i ∑ fu 2
i i

= 10 = −3 =9
h2 102
Now, Variance σ 2 =
N 2{N ∑ f u
i i
2
− ( ∑ f i i )
u 2
=
10 2} { 2
10 × 9 − ( 3) = 81 . }
∴ standard Deviation σ = 81 = 9
2 + 4 + 10 + 12 + 14 + x1 + x2
49. Ans. (a) Let the remaining two observations be x1 and x2 . Mean, x =
7
7

42 + x1 + x2 ∑x 2
i

()
2
⇒8= ⇒ x1 + x2 = 14 ….(i) ∵ x = 8 ( given )  And variance, σ 2 = i =1
− x
7 7
n
 
 ∑x 2
i
2 22 + 4 2 + 102 + 12 2 + 142 + x12 + x22
− 8 ∵σ 2 = 16 ( given ) 
2
∵ we know that σ 2 = i =1
− x  ⇒ 16 =
 n  7
 
⇒ 560 = 4 + 16 + 100 + 144 + 196 + x12 + x22 ⇒ x12 + x22 = 100 ….(ii)
= 2 ( x12 + x22 ) ⇒ ( x1 − x2 ) = 2 ( x12 + x22 ) − ( x1 + x2 ) = 2 (100 ) − 142 = 4
2 2 2 2
Now, ( x1 + x2 ) + ( x1 − x2 )
⇒ x1 − x2 = ± 2 ….(iii). Case I : x1 − x2 = 2, x1 + x2 = 14 . Case II : x1 − x2 = −2, x1 + x2 = 14 .

th
Office.: 606 , 6 Floor, Hariom Tower, Circular Road, Ranchi-1, Ph.: 0651-2562523, 9835508812, 8507613968
8 BY R. K. MALIK’S NEWTON CLASSES
( )
Solving these equations, we get : x1 = 6, x2 = 8
50. Ans. (d) For the old sequence : Number of observations, n = 9 : Mean, x = 43; Standard Deduction,
n 9

∑ xi ∑x i 9
σ = 5 . We have : x = i =1
= i =1
⇒ ∑ xi = 9 x = 9 × 43 = 387 ….(i). Also,
n 9 i =1
n 9 9

∑x 2
i ∑x 2
i ∑x 2
i 9

()
2
− 1849 ⇒ ∑ xi2 = 16866 ….(ii)
2
σ2 = i =1
− x = i =1
− ( 43) ⇒ 52 = i =1

n 9 9 i =1
10 9

∑ x ∑ x + 63
i i
For the new sequence : Number of observations, n′ = 10 ; Mean x′ = i =1
= i =1

10 10
387 + 63 450 10 9
=
10
=
10
= 45 ; Also, ∑x =∑x
i =1
2
i
i =1
2
i + 632 = 16866 + 3969 = 20835 . ∴Standard Deviation
1/ 2
 10 2 
 ∑ xi   20835 
1/ 2

σ ′ =  i =1 − x′  ( ) = − 452  = 58.5 = 7.65


 10   10 
 
x + ( x + 2 ) + ( x + 4) + ( x + 6 ) + ( x + 8) 5 x + 20
51. Ans. (b) As given = 11 ⇒ = 11 ⇒ x = 7 .
5 5
11 + 13 + 15
∴ Mean of the last three values = = 13
3
52. Ans. (a) We have
x f cf
7 2 2
8 1 3
9 5 8
10 4 12
11 6 18
12 1 19
13 3 22

median =
(11th + 12th ) value = 9
2
53. Ans. (a) Let correct value of the number = a . Then from given information sum of numbers
= 10 × 7 = 70 . ∴ 60 − 9 + a = 70 ⇒ a = 19
54. Ans. (b) Maximum number of firms is 1200 in the group of 1200 – 1800. Therefore highest bar would
correspond to 1200 – 1800 class.
55. Ans. (b) The cumulative frequency table is as follows :

x 2 3 4 5 6
cf 2 6 14 18 19
th
 N +1  N +1 th
Here N = 19. ∴ Q1 =   th value = 5th value = 3 . Q3 = 3   value = 15 value = 5 .
 4   4 
Q − Q1 5 − 3
∴Q = 3 = =1
2 2
56. Ans. (b) Sum of the values = 10 × 6 = 60 . Sum of four values = 4 × 7.5 = 30 . Sum of remaining 6
th
Office.: 606 , 6 Floor, Hariom Tower, Circular Road, Ranchi-1, Ph.: 0651-2562523, 9835508812, 8507613968
( ) BY R. K. MALIK’S NEWTON CLASSES 9
30
values = 60 − 30 = 30 . Their mean = = 5.
6
57. Ans. (a) Writing values in ascending order : 7, 10, 12, 15, 17, 19, 25.
7 +1
Q1 = th value = Second value = 10 ,
4
 7 +1  Q3 − Q1 19 − 10
Q3 = 3   th value = sixth value = 19 . ∴ Q = = = 4.5 .
 4  2 2
58. Ans. (c) Using standard formula for finding the combined mean we have combined
10 ( 7 ) + 5 ( 3 )
Mean = = 8.5
7+3
59. Ans. (a)

Class mid value fi xi − 25 fiui fiui 2


ui =
10
0 – 10 5 1 −2 −2 4
10 – 20 15 3 −1 −3 3
20 – 30 25 4 0 0 0
30 – 40 35 2 1 2 2
10 -3 9
2
 f u2 
σ = h ∑ i i
 ∑ fi ui  9 −3 
2
2 2
−    = 100  −    = 100[ 0.9 − 0.09] = 81 ∴σ = 9
 N
  N  
 10  10  
nx − x p + x′p
60. Ans. (b) Sum of the observations = nx , Hence new sum = nx − x p + x′p . ∴ new mean =
n
61. Ans. (b) Given information is Total number of students = 100 . Number of boys students = 70 .
Number of girls
students = 30 . Also let the average marks of the girls = x .
Given : 70 × 750 + 30 × x = 720 × 100 ⇒ 52500 + 30 x = 7200 ⇒ 30 x + 72000 − 52500 = 19500
19500
⇒x= = 650
30
3 + 4 + x + 7 + 10
62. Ans. (c) Given information ⇒ = 6 ⇒ 24 + x = 30 ⇒ x = 30 − 24 = 6
5
63. Ans. (a) Here, there are six items, therefore, median is the mean of 3rd and 4th observations.
16 + 18
= = 17
2
64. Ans. (c) Since, 6 is repeated maximum number of times, therefore, mode = 6 .
1.5 + 2.4 + 3.6 + 4. f
65. Ans. (d) Since mean = 3 . ∴ = 3 ⇒ 5 + 8 + 18 + 4 f = 3 (15 + f )
5+ 4+6+ f
⇒ 31 + 4 f = 45 + 3 f ⇒ f = 14
66. Ans. (c) Sum of total number = 18 × 5 = 90 . After one number excluded. Sum of total
number = 16 × 4 = 64 . Then, excluded number is 90 − 64 = 26 .
67. Ans. (c) In this case each value of the variable is increased by same constant 10, therefore, the
standard deviation
remains unchanged.
68. Ans. (a) If each item of a data is increased ( or decreased ) by the same quantity, the standard
deviation ( and the variance ) is not changed.
1/ n
69. Ans. (c) ( x1 , x2 ....xn ) is G.M.
70. Ans. (c) Total weight of 7 students is = 55 × 7 = 385 kg. and Sum of weight of 6 students
= 52 + 58 + 55 + 53 + 56 + 54 = 328 kg. ∴ Weight of seventh student = 385 − 328 = 57 kg.

th
Office.: 606 , 6 Floor, Hariom Tower, Circular Road, Ranchi-1, Ph.: 0651-2562523, 9835508812, 8507613968
10 ( ) BY R. K. MALIK’S NEWTON CLASSES
3 1
71. Ans. (c), we know that Mode = 3 Median −2 Mean ⇒ Mean = Median −
2 2
1
Mode = ( 3 Median − Mode )
2
72. Ans. (c) S.D. ( and variance ) of a data is not changed when each observation is increased ( or
decreased ) by the same constant
73. Ans. (d) If each item of a data is increased or decreased by the same constant, the standard deviation
of the data remains unchanged.
74. Ans. (c) The runs scored by two batsmen in their last ten matches are as follws
Batsman A : 30, 91,0, 64, 42, 80, 30,5,117,71 Batsman B : 53, 46,48,50,53, 53,58, 60, 57,52
30 + 91 + 0 + 64 + 42 + 80 + 30 + 5 + 117 + 71 530
Mean for batsman A = = = 53
10 10
53 + 46 + 48 + 50 + 53 + 53 + 58 + 60 + 57 + 52 53
Mean for batsman B = = = 53
10 10
Also , the median is obtained by first arranging the data in ascending or descending order and applying
the rules
To apply the formula to obtain median first arrange the data in ascending order
For batsman A 0 5 30 30 42 64 71 80 91 117
For batsman B 46 48 50 52 53 53 53 57 58 60
Hence we have n = 10 which is even number . So median is the mean of 5th and 6th observations .
42 + 64 106 53 + 53 106
Median for batsman A = = = 53 Median for batsman B = = = 53
2 2 2 2
75. Ans. (d)
76. Ans. (c), Range = Maximum value – Least value = 80 – 8 = 72
77. Ans. (b), Range = 9 – 2 = 7
Sum of observations 3 + 4 + x + 7 + 10
78. Ans. (c), Mean = ⇒6 = ⇒ x = 30 − 24 = 6
Number of observations 5
79. Ans. (d), 360 = 233251 = 2 a 3b5c Number of odd divisors of 360 = ( b + 1)( c + 1) = 3 × 2 = 6
and odd divisor are 1,3,5,9,15,45
Sum of all odd divisors 1 + 3 + 5 + 9 + 15 + 45 = 78
Sum of odd divisors 78
Mean of odd divisors = = = 13
Number of odd divisors 6
Valueof 3rd divisor + Value of 4 th divisor 5+9
Now median value of all the odd divisor = = =7
2 2
13 + 7
∴ Median = 7 ∴ Mean of "median and mean of odd divisors of 360" = = 10
2
80. Ans. (c), Let the n numbers are x1 , x2 , x3 ,....xn ∴ New item are x1 − 4, x2 − 4, x3 − 4,.....xn − 4

∴ New mean =
( x1 − 4 ) + ( x2 − 4 ) + ..... + ( xn − 4 ) =
x1 + x2 + ....xn − 4n x1 + x2 + ... + xn
= −4= x −4
n n n
So, new mean is reduced by 4 unit in x
n ( n + 1)
81. Ans. (a),As we know that sum of first n natural = Hence Sum of first ( n + 1) natural numbers
2
=
( n + 1)( n + 2 ) ∴ Required Mean = ( n + 1)( n + 2 ) = n + 2
2 2 ( n + 1) 2

th
Office.: 606 , 6 Floor, Hariom Tower, Circular Road, Ranchi-1, Ph.: 0651-2562523, 9835508812, 8507613968
( ) BY R. K. MALIK’S NEWTON CLASSES 11
n
82. Ans. (c), As xi − x = d and sum of all deviations about the mean x denoted by ∑(x − x )
i =1
i is always
n
equal to zero. ∴ ∑ ( xi − x ) = 0
i =1

83. Ans. (a), We have,


Class x f fx
0-10 5 8 40
10-20 15 10 150
20-30 25 f1 25 f1
30-40 35 15 525
40-50 45 f2 45 f 2
Also, ∑ f = 70, ∑ fx = 25 f + 45 f + 715 ⇒ 8 + 10 + f + 15 + f = 70 ⇒ f + f
i 2 1 2 1 2 = 37 ..... ( i )

and x =
∑ fx ⇒ 5 f + 9 f = 277 ..... ( ii ) Solving (i) and (ii), we get f = 14 and f 2 = 23
∑f
1 2 1

84. Ans. (a), Mean =


(1× 6 ) + ( 7 × 2 ) + (1.9 × k ) + ( 9 × 3) + (10 × 4 ) ⇒ 3.6 =
87 + 1.9k
⇒ k = 30
1+ 2 + k + 3 + 4 k + 10

85. Ans. (b), A.M. =


( ax1 + b ) + ( ax2 + b ) + ... + ( axn + b ) =
a ( x1 + x2 + ... + xn ) + nb
n n
a ( x1 + x2 + ... + xn )
= + b = ax + b
n
86. Ans. (a), We have,
Mid point (x) Frequency (f) fx
10 17 170
30 f1 30 f1
50 32 1600
70 f2 70 f 2
90 19 1710
Total 120 30 f1 +70 f 2 +3480
1 1
Now, x =
120
∑ fx ⇒ 50 =
120
× ( 30 f1 + 70 f 2 + 3480 ) ⇒ 600 = 3 f1 + 7 f 2 + 348

⇒ 3 f1 + 7 f 2 = 252 ..... ( i ) Also, f1 + f 2 + 68 = 120 ⇒ f1 + f 2 = 52 ..... ( ii )


Solving (i) and (ii), we get f1 = 28, f 2 = 24
87. Ans. (b), Incorrect sum of all items = 78.4 × 25 = 1960 kg ∴ Correct sum = 1960 − 69 + 96 = 1987 kg
1987
∴ Correct average = = 79.48 kg.
25
(27 + x) + (31 + x) + (89 + x) + (107 + x) + (156 + x)
88. Ans. (b), = 82 ⇒ 5 x + 410 = 410 ⇒ 5 x = 0 ⇒ x = 0
5
(130 + x) + (126 + x) + (68 + x) + (50 + x) + (1 + x)
∴ Required mean =
5
130 + 126 + 68 + 50 + 1 375
= = = 75.
5 5

th
Office.: 606 , 6 Floor, Hariom Tower, Circular Road, Ranchi-1, Ph.: 0651-2562523, 9835508812, 8507613968
12 ( BY R. K. MALIK’S NEWTON CLASSES
)
89. Ans. (d), Total weight of 35 students = 35 × 40 = 1400 kg. According to question,
 1 81
Weight of ( 35student + teacher ) = 36 ×  40 +  = 36 × = (18 × 81) = 1458 kg
 2 2
So, weight of teacher = 1458 − 1400 = 58 kg.
90. Ans. (a), Let n(G) and n(B) be the number of girls and boys in a class respectively and m(G) and m(B)
be the total of marks scored by girls and boys respectively.
m (G ) + m ( B )
∴ = 53 ..... ( i ) Also, m ( G ) = 55n ( G ) and m ( B ) = 50n ( B ) ..... ( ii ) . Form (i) and (ii),
n (G ) + n ( B )
55n ( G ) + 50n ( B ) 2 n (G )
we get = 53 ⇒ n ( B ) = n ( G ) ∴ Percentage of girls × 100
n (G ) + n ( B ) 3 n ( B ) + n (G )
n (G )
= ×100 = 60%
2 
 + 1 n ( G )
3 
91. Ans. (d), Let number of males = x and number of females = y. According to question,
206 ( x + y ) = 210 x + 190 y ⇒ 206 x + 206 y = 210 x + 190 y ⇒ 4 x = 16 y ⇒ x = 4 y So % of females
y y 1
employed in the factory = × 100 = × 100 = × 100 = 20%
x+ y 5y 5
a + a + 1 + a + 2 + ... + a + 6
92. Ans. (d), = m ⇒ 7a + 21 = 7m ⇒ a + 3 = m ..... ( i )
7
(a + 2) + (a + 3) + ... + (a + 12)
Now A.M. of 11 consecutive integers starting form a + 2 is
11
11a + 77
= = a + 7 = a + 3 + 4 = m + 4 (using (i))
11
93. Ans. (c), Sum of 9 items = 15 × 9 = 135 , Sum of 10 items = 16 ×10 = 160
∴ Value of added term = 160 − 135 = 25
1 + 2 + 3 + ... + 99 + x 99.100 50
94. Ans. (c), = 100 x ⇒ 10000 x = +x⇒x=
100 2 101
95. Ans. (c), Let the average after 16th innings be x Then, average after 15th innings = ( x − 2 )
∴ Given information ⇒ 15 ( x − 2 ) + 70 = 16 x ⇒ x = 40
96. Ans. (c), Range = 72 − 27 = 45
97. Ans. (a),
98. Ans. (a), Arranging the given data in ascending order, we get 2, 3, 5, 7, 8, 9, 10, 11, 14, Here n = 9,
1 th 1 th
which is odd. ∴ Median = ( n + 1) observation = ( 9 + 1) observation = 5th observation = 8 Thus,
2 2
M = 8. The values of ( xi − M ) are:- −6, − 5, − 3, − 1, 0,1, 2,3, 6.
9
∴ ∑ xi − M = ( 6 + 5 + 3 + 1 + 0 + 1 + 2 + 3 + 6 ) = 27
i =1
9

∑ x −M
27i
⇒ M .D. ( M ) = i =1
= 3 Hence, M.D. (M) = 3.
=
9 9
99. Ans. (d), First we find the median.

th
Office.: 606 , 6 Floor, Hariom Tower, Circular Road, Ranchi-1, Ph.: 0651-2562523, 9835508812, 8507613968
( BY R. K. MALIK’S NEWTON CLASSES
) 13
Class Frequency ( f i ) Cumulative Frequency (C) Mid value ( xi )
95-105 9 9 100
105-115 13 22 110
115-125 25 47 120
125-135 30 77 130
135-145 13 90 140
145-155 10 100 150
N = ∑ f i = 100
N
Thus N = 100 and therefore, = 50 ⇒ median class is 125 − 135. ⇒ l = 125, f = 30, h = 10 and
2
N 
 −C
C = 47 ∴ Median M = l + 
2  × h = 125 + ( 50 − 47 ) ×10  = 126 ∴ M = 126.
 
f  30 
Now, we prepare the table given blow.
fi xi xi − M f i xi − M
9 100 26 234
13 110 16 208
25 120 6 150
30 130 4 120
13 140 14 182
10 150 24 240
N = 100 1134

∴ M.D. (M) =
∑f 1134
i xi − M
= 11.34 =
N 100
100. Ans. (b), Converting the given inclusive data into an exclusive data, we prepare the table given below.
Class Frequency ( f i ) C Mid-value ( xi )
15.5-20.5 5 5 18
20.5-25.5 6 11 23
25.5-30.5 12 23 28
30.5-35.5 14 37 33
35.5-40.5 26 63 38
40.5-45.5 12 75 43
45.5-50.5 16 91 48
50.5-55.5 9 100 53
N = 100
N
Thus N = 100 and therefore, = 50 ⇒ Median class is 35.5 – 40.5 ⇒ l = 35.5, f = 26, h = 5 and
2
N 
 −C
C = 37 ∴ Median = l + 
2  × h = 35.5 + ( 50 − 37 ) × 5 = 35.5 + 2.5 = 38. Thus, M = 38
  ( )
f  26 
Now, we prepare the table given below.

th
Office.: 606 , 6 Floor, Hariom Tower, Circular Road, Ranchi-1, Ph.: 0651-2562523, 9835508812, 8507613968
14 ( ) BY R. K. MALIK’S NEWTON CLASSES
fi xi xi − M f i xi − M
5 18 20 100
6 23 15 90
12 28 10 120
14 33 5 70
26 38 0 0
12 43 5 60
16 48 10 160
9 53 15 135
N = 100 735

Thus, ∑f xi − M = 735 and N = 100 ∴ M.D. (M) =


∑f i xi − M
=
735
= 7.35. Hence, the mean
i
N 100
deviation about the median is 7.35.
6 + 7 + 10 + 12 + 13 + 4 + 8 + 12 72
101. Ans. (a), Mean of the given data is x = = =9
8 8
8

∑ x −x i
The value of the deviations, i.e., xi − x are 3, 2, 1, 3, 4, 5, 1, 3 ∴ M.D. ( x ) = i =1

8
3 + 2 + 1 + 3 + 4 + 5 + 1 + 3 22
= = 2.75
8 8
1 20 200
102. Ans. (c), The mean ( x ) of the given data is, x = ∑
20 i =1
xi =
20
= 10 , The values of the deviations

form mean, i.e., xi − x are 2, 7, 8, 7, 6, 1, 7, 9, 10, 5, 2, 7,8,7, 6, 1, 7, 9, 10, 5


20
124
∴ ∑ xi − x = 124 and M .D. ( x ) = = 6.2
i =1 20
103. Ans. (a), Here the number of observations is 11 which is odd. Arranging the data into ascending order,
th
 11 + 1  th
we have 3, 3, 4, 5, 7, 9, 10, 12, 18, 19, 21 Now, Median =   observation or 6 observation
 2 
= 9. The absolute values of the respective deviation from the median are 6, 6, 5, 4, 2, 0, 1, 3, 9, 10, 12
11
1 11 1
Therefore ∑ xi − M = 58 and M .D. ( M ) = ∑ xi − M = × 58 = 5.27
i =1 11 i =1 11
104. Ans. (c), Let us make a table of the given data.
xi fi f i xi xi − x f i xi − x
2 2 4 5.5 11
5 8 40 2.5 20
6 10 60 1.5 15
8 7 56 0.5 3.5
10 8 80 2.5 20
12 5 60 4.5 22.5
Total 40 300 92
6 6
1 6
1 6
Here N = ∑ f i = 40, ∑ fi xi = 300 , Therefore x = ∑ f i xi = × 300 = 7.5 . Now, ∑f i xi − x = 92
i =1 i =1 N i =1 40 i =1

th
Office.: 606 , 6 Floor, Hariom Tower, Circular Road, Ranchi-1, Ph.: 0651-2562523, 9835508812, 8507613968
( BY R. K. MALIK’S NEWTON CLASSES
) 15
1 6
1
∴ M .D. ( x ) = ∑ fi xi − x = × 92 = 2.3
N i =1 40
105. Ans (a), The given observations are already in ascending order. Adding a row corresponding to
cumulative frequencies to the given, we get
xi 3 6 9 12 13 15 21 22
fi 3 4 5 2 4 5 4 3
c.f. 3 7 12 14 18 23 27 30
Now, N = 30, which is even. ∴ Median is the mean of the 15th and 16th observation. Both of the
these observations lie in the cumulative frequency 18, for which the corresponding observation is 13.
15th observation + 16 th observation 13 + 13
∴ Median M = = = 13
2 2
Now,
xi − M 10 7 4 1 0 2 8 9

fi 3 4 5 2 4 5 4 3
f i xi − M 30 28 20 2 0 10 32 27
8 8
1 8
1
We have, ∑
i =1
f i = 30 and ∑
i =1
f i xi − M = 149 . Therefore M.D. ( M ) =
N
∑f
i =1
i xi − M =
30
×149 = 4.97

106. Ans. (b), We make the following table from the given data
Marks obtained Number of student Mid-point xi f i xi xi − x f i xi − x
10-20 2 15 30 30 60
20-30 3 25 75 20 60
30-40 8 35 280 10 80
40-50 14 45 630 0 0
50-60 8 55 440 10 80
60-70 3 65 195 20 60
70-80 2 75 150 30 60
1800 400
7 7 7
1 7
1800
Here, N = ∑ f i = 40, ∑ f i xi = 1800, ∑ fi xi − x = 400 . Therefore x = ∑fx i i = = 45
i =1 i =1 i =1 N i =1 40
1 7
1
and M .D. ( x ) =
N
∑f
i =1
i xi − x =
40
× 400 = 10

107. Ans. (a), Form the following table from the given data
Class Frequency f i Cumulative frequency (c.f.) Mid-points xi xi − M f i xi − M
0-10 6 6 5 23 138
10-20 7 13 15 13 91
20-30 15 28 25 3 45
30-40 16 44 35 7 112
40-50 4 48 45 17 68
50-60 2 50 55 27 54
50 508

th
Office.: 606 , 6 Floor, Hariom Tower, Circular Road, Ranchi-1, Ph.: 0651-2562523, 9835508812, 8507613968
16 BY R. K. MALIK’S NEWTON CLASSES
( )
N
−C
N
Here N = 50 ∴ = 25 Therefore, 20-30 is the median class. Median = l + 2 ×h
2 f
25 − 13
Here l = 20, C = 13, f = 15, h = 10 and N = 50 Therefore, Median = 20 + × 10 = 20 + 8 = 28
15
1 6
1
Thus, Mean deviation about median is given by M.D. (M) =
N
∑f
i =1
i xi − M =
50
× 508 = 10.16

x + ( x + y ) + ( x + 2 y ) + .... + ( x + 2ny )
108. Ans. (a), Here x =
2n + 1

=
( 2n + 1) [ x + x + 2ny ] = x + ny (∵ Total number of observations = 2n + 1)
2 ( 2n + 1)
xi d = xi = x
x ny
x+ y ( n − 1) y
x + 2y ( n − 2) y
x + 3y ( n − 3) y
⋮ ⋮
⋮ ⋮
x + ny 0
x + ( n + 1) y y

⋮ ⋮
⋮ ⋮
x + 2ny ny

∑d = 2  ny + ( n − 1) y + ... + y  =
2 ⋅ y n ( n + 1)
= ny ( n + 1) ∴ M.D. ( x ) =
∑d =
ny ( n + 1)
2 2n + 1 2n + 1

109. Ans. (a), Mean x =


∑fx i i
=
3 + 9 + 20 + 98 + 63 + 44 + 39 + 60 336
= =8
∑f i 42 42

M.D. ( x ) =
∑fx i i
=
3 ( 7 ) + 3 ( 5) + 4 ( 3) + 14 (1) + 7 (1) + 4 ( 3) + 3 ( 5) + 4 ( 7 )
∑f i 42
21 + 15 + 12 + 14 + 7 + 12 + 15 + 28 62
= = = 2.95
42 21
2 + 9 + 9 + 3 + 6 + 9 + 4 42
110. Ans. (b), Mean x = = =6 M.D. ( x ) =
∑ x −x
i

7 7 n
4 +3+ 3+3+ 0+ 3+ 2
= = 2.57
7

th
Office.: 606 , 6 Floor, Hariom Tower, Circular Road, Ranchi-1, Ph.: 0651-2562523, 9835508812, 8507613968
( ) BY R. K. MALIK’S NEWTON CLASSES 17
111. Ans. (a),
xi fi f i xi xi − x f i xi − x
1 3 3 11 33
4 4 16 8 32
9 5 45 3 15
12 2 24 0 0
13 4 52 1 4
14 5 70 2 10
21 4 84 9 36
22 3 66 10 30
Total 30 360 160

Mean ( x ) =
∑fx i i
=
360
= 12 M.D. ( x ) =
∑ f x −x
i i
=
160
= 5.33 .
∑f i 30 ∑f i 30
112. Ans. (b), Taking assumed mean a = 35, we construct the
Class Class marks xi fi xi − 35 f i ui xi − x f i xi − x
ui =
10
0-10 5 4 −3 −12 30 120
10-20 15 6 −2 −12 20 120
20-30 25 10 −1 −10 10 100
30-40 35 20 0 0 0 0
40-50 45 10 1 10 10 100
50-60 55 6 2 20 20 120
60-70 65 4 3 30 30 120
Total 60 680

x = 35 +
∑ f u ×10 = 35 +  0 ×10  = 35,
i i
M.D. (about mean) =
∑ f x −x
i i
=
680
= 11.33 .
 
∑f i  60  ∑f i 60
113. Ans. (a), The scores (arranged in ascending order) are 34, 38, 42, 44, 46, 48, 54, 55, 63, 70. Since, the
th th
 10   10 
number of observation is 10. ∴ Median (M) = mean of   and  + 1 observations
 2  2 
46 + 48
= mean of 5th and 6th observations = = 47 .
2
xi 34 38 42 44 46 48 54 55 63 70 Total
xi − M 13 9 5 3 1 1 7 8 16 23 86
10

86 ∑ x −M i
Hence, mean deviation about median = = 8.6 i =1
=
10 10
114. Ans. (b), Let the other two observations be a, b. Then, the five observations are 1, 2, 6, a, b.
a + b +1+ 2 + 6
We have, mean x = 4 ⇒ = 4 ⇒ a + b = 11
5
2
∑(x − x )
2 2 2 2 2

Also, σ 2
= i
= 5.2 ⇒
(1 − 4 ) + ( 2 − 4 ) + ( 6 − 4 ) + ( a − 4 ) + ( b − 4 ) = 5.2
n 5

th
Office.: 606 , 6 Floor, Hariom Tower, Circular Road, Ranchi-1, Ph.: 0651-2562523, 9835508812, 8507613968
18 ( ) BY R. K. MALIK’S NEWTON CLASSES
2 2
⇒ ( a − 4) + (b − 4) = 26 − 9 − 4 − 4 = 9 ⇒ a 2 + 16 − 8a + b 2 + 16 − 8b = 9 ⇒ a 2 + b 2 − 8 ( a + b ) = −23

⇒ a 2 + b 2 − 88 = −23 (∵a + b = 11) ⇒ a 2 + b 2 = 65 ⇒ ( a + b ) − 2ab = 65 ⇒ 2ab = 121 − 65


2

⇒ ab = 28 Solving a + b = 1 and ab = 28 we get a = 7, b = 4, or a = 4, b = 7


115. Ans. (c), First 10 multiples of 3 are 3, 6, 9, 12,….30. This is an A.P. with a = 3, n = 10 and l = 30
n 10 165
∴ Sum = ( a + l ) = ⋅ ( 3 + 30 ) = 165 ∴ Mean, x = = 16.5 .
2 2 10

116. Ans. (d), Variance, σ 2


=
∑ x −(x)
2
i 2  32 + 62 + 92 + ... + 302
=
2
− (16.5) 
n  10 
32 × {12 + 22 + 32 + ... + 10 2 } 2 9 × 10 × (10 + 1)( 2 × 10 + 1) 2
= − (16.5 ) = − (16.5 )
10 6 × 10
 9 × 10 × 11× 21   693 
= − 272.25  =  − 272.25  = ( 346.5 − 272.25 ) = 74.25 .
 6 × 10   2 
117. Ans. (a), We may prepare the table given below.
xi fi f i xi ( xi − x ) ( xi − x )
2
f i ( xi − x )
2

10 3 30 −8 64 192
15 2 30 −3 9 18
18 5 90 0 0 0
20 8 160 2 4 32
25 2 50 7 49 98
Total 360 340

x = Mean =
∑fx i i
=
360 5
= 18 , N = ∑ f i = 20,
5

∑ f (x − x)
2
= 340
∑f i 20 i =1 i =1
i i

1  1 
⋅ ∑ f i ( xi − x ) =  × 340  = 17 .
2
∴ Variance, σ 2 =
N  20 
118. Ans. (a), Converting the given series into an exclusive series, we prepare the table given below.
Class Frequency ( f i ) Mid-point ( xi )
yi = i
( x − 42.5 ) yi2 f i yi f i yi2
4
32.5-36.5 15 34.5 −2 4 −30 60
36.5-40.5 17 38.5 −1 1 −17 17
40.5-44.5 21 42.5 = A 0 0 0 0
44.5-48.5 22 46.5 1 1 22 22
48.5-52.5 25 50.5 2 4 50 100
N = 100 25 199

∴ A = 42.5, h = 4, N = ∑ f i = 100, ∑ f i yi = 25 and ∑fy 2



= 199 . ∴ x =  A +
∑ fi yi × h 
i i
N 
 
 25 
⇒ x =  42.5 + × 4  = 43.5 . Thus, mean = 43.5
 100 

119. Ans. (a), Variance, σ 2 =


h2
N 2 (
⋅ N ⋅ ∑ fi yi2 − ( ∑ f i yi )
2
) = 10000
16
× {100 × 199 − ( 25 ) }
2

th
Office.: 606 , 6 Floor, Hariom Tower, Circular Road, Ranchi-1, Ph.: 0651-2562523, 9835508812, 8507613968
( BY R. K. MALIK’S NEWTON CLASSES
) 19
16  16  3084
= × (19900 − 625 ) =  × 19275  = = 30.84
10000  10000  100
120. Ans. (c), Standard deviation, σ = 30.84 = 5.55 .
121. Ans. (b), Let the observations be x1 , x2 ,....., x20 and x be their mean.
Given that variance = 5 and n = 20 .
1 20 1 20 20
We know that, Variance (σ 2 ) = ∑ ∑ ∑( x − x )
2 2 2
( xi − x ) , i.e.,5 = ( xi − x ) or i = 100 …..(i)
n i =1 20 i =1 i =1

If each observation is multiplied by 2, and the new resulting observation are yi , then
1 1 20 1 20 1 20 1
yi = 2 xi i.e., xi = yi Therefore, y = ∑ yi = ∑ 2 xi = 2 ⋅ ∑ xi i.e., y = 2 x or x = y
2 n i =1 20 i =1 20 i =1 2
2
1 1 
20 20
Substituting the value of xi and x in (i), we get ∑  yi − y  = 100, i.e., ∑ ( yi − y ) = 400
2

i =1  2 2  i =1

1
Thus the variance of new observations = × 400 = 20
20
122. Ans. (a), Let the other two observation be x and y. Therefore, the series is 1, 2, 6, x, y.
1+ 2 + 6 + x + y
Now, Mean ( x ) = 4.4 = or 22 = 9 + x + y Therefore, x + y = 13 ……(i)
5
1 5

2
Also, variance = 8.24 = ( xi − x ) i.e.,
n i =1
1 2 2 2 2
8.24 = ( 3.4 ) + ( 2.4 ) + (1.6 ) + x 2 + y 2 − 2 × 4.4 ( x + y ) + 2 × ( 4.4 )  or
5 
41.20 = 11.56 + 5.76 + 2.56 + x 2 + y 2 − 8.8 × 13 + 38.72 . Therefore x 2 + y 2 = 97 ..... ( ii )
But from (i), we have x 2 + y 2 + 2 xy = 169 ..... ( iii ) From (ii) and (iii), we have 2 xy = 72 ...... ( iv )
2
Subtracting (ii) and (iii),we get x 2 + y 2 − 2 xy = 97 − 72 i.e., ( x − y ) = 25 or x − y = ± 5 ..... ( v ) So,
from (i) and (v), we get x = 9 y = 4 when x − y = 5 or x = 4, y = 9 when x − y = −5 Thus, the
remaining observations are 4 and 9
1 n

2
123. Ans. (a), Let x be the mean of x1 , x2 ,...xn . Then the variance is given by σ 12 = ( xi − x ) . If ‘a’ is
n i =1
added to each observation, the new observations will be yi = xi + a ..... ( i )
1 n 1 n
Let the mean of the new observation be y . Then y = ∑ i n∑
n i =1
y =
i =1
( xi + a )
1 n n
 1 n na
=  ∑
n  i =1
xi + ∑ a
i =1 
 = ∑ xi +
n i =1 n
= x + a i.e., y = x + a ..... ( ii ) Thus, the variance of the new

1 n 1 n 1 n
∑ ∑ ∑
2 2 2
observations σ 22 = ( yi − y ) = ( xi + a − x − a ) [Using (i ) and (ii )] σ 2
2 = ( xi − x ) = σ 12
n i =1 n i =1 n i =1
Thus, the variance of the new observations is same as the of the original observations.
124. Ans. (a), To find the standard deviation, we construct the following, table, taking assumed mean
a = 48 .

th
Office.: 606 , 6 Floor, Hariom Tower, Circular Road, Ranchi-1, Ph.: 0651-2562523, 9835508812, 8507613968
20 ( ) BY R. K. MALIK’S NEWTON CLASSES
xi d i = xi − a d i2
34 −14 196
38 −10 100
42 −6 36
44 −4 16
46 −2 4
48 0 0
54 6 36
55 7 49
63 15 225
70 22 484
Total 14 1146
2

∴ σ ( S.D.) =
∑d i
2
 ∑ di  1146  14 
2

−   = −   = 114.6 − 1.96 = 112.64 = 10.61 nearly.


n  n  10  10 
125. Ans. (c), Given that number of observation (n) = 100, Incorrect mean ( x ) = 40
1 n 1 100 100
We know that x = ∑ i
n i =1
x i.e., 40 = ∑ xi or
100 i =1
∑x
i =1
i = 4000 .This is incorrect sum of all observations

3990
Hence correct sum of all observation = 4000 −50 + 40 = 3990 Hence Correct Mean = = 39.9
100
2
1 n 2 1  n  1 n 2
∑ xi − n2  ∑ ∑
2
126. Ans. (d), Also, standard deviation (σ ) = xi  = xi − ( x ) i.e.
n i =1 i =1  n i =1

1 n
1 n
× Incorrect ∑ xi2 − ( 40 ) × Incorrect ∑ xi2 − 1600
2
5.1 = or 26.01 =
100 i =1 100 i =1

n
Therefore, Incorrect ∑ xi2 = 100 ( 26.01 + 1600 ) = 162601 Now,
i =1
n n
Correct ∑ xi2 = Incorrect ∑ xi2 ( 50 ) + ( 40 ) = 162601 − 2500 + 1600 = 161701
2 2

i =1 i =1

Correct ∑ xi2 2
Therefore correct standard deviation = − ( Correct mean )
n
161701 2
− ( 39.9 ) = 1617.01 − 1592.01 = 25 = 5
100

th
Office.: 606 , 6 Floor, Hariom Tower, Circular Road, Ranchi-1, Ph.: 0651-2562523, 9835508812, 8507613968
( BY R. K. MALIK’S NEWTON CLASSES
) 21
127. Ans. (a), From the given data we can form the following table. Let assumed mean A = 14
xi x − 14
di = i ( xi − x ) ( xi − x )2
2
6 −4 −9 81
8 −3 −7 49
10 −2 −5 25
12 −1 −3 9
14 0 −1 1
16 1 1 1
18 2 3 9
20 3 5 25
22 4 7 49
24 5 9 81
Total 5 330
n

∑d i
5 1 10
× 2 = 15 and Variance (σ 2 ) = ∑ ( xi − x ) × 330 = 33
2
Therefore, Mean x = A + i =1
× h = 14 +
n 10 n i =1
128. Ans. (a), Form the data in tabular from, we get
xi fi f i xi xi − x ( xi − x )
2
f i ( xi − x )
2

4 3 12 −10 100 300


8 5 40 −6 36 180
11 9 99 −3 9 81
17 5 85 3 9 45
20 4 80 6 36 144
24 3 72 10 100 300
32 1 32 18 324 324
30 420 1374
7

7 7 ∑fx i i
1
N = 30, ∑ f i xi = 420, ∑
2
f i ( xi − x ) = 1374 Therefore x = i =1
= × 420 = 14
i =1 i =1 N 30
1 7
1
Hence variance (σ 2 ) = ∑ f (x − x)
2
i i = × 1374 = 45.8
N i =1 30
129. Ans. (a), Form the given data, we construct the following table
Class Frequency ( f i ) Mid-point ( xi ) f i xi (x − x)
2
i f i ( xi − x )
2

30-40 3 35 105 729 2187


40-50 7 45 315 289 2023
50-60 12 55 660 49 588
60-70 15 65 975 9 135
70-80 8 75 600 169 1352
80-90 3 85 255 529 1587
90-100 2 95 190 1089 2178
Total 50 3100 10050

th
Office.: 606 , 6 Floor, Hariom Tower, Circular Road, Ranchi-1, Ph.: 0651-2562523, 9835508812, 8507613968 21
22 ( BY R. K. MALIK’S NEWTON CLASSES
)
1 7
3100
Thus, Mean x = ∑ f i xi = = 62
N i =1 50
1 7
1
130. Ans. (c), Variance (σ 2 ) = ∑ f (x − x)
2
i i = × 10050 = 201
N i =1 50
131. Ans. (d), Standard deviation (σ ) = 201 = 14.18
132. Ans. (d), S.D. of a series unaltered if each item is raised (reduced) by the some constant i.e., S.D.is
independent of change of origin. ∴ New S.D. = 30
99 99 99
133. Ans. (b), Variance of 1, 2, 3, 4, 5, …. ,10 is . ∴ Variance of 3, 6, 9, 12,…...,30 is ( 32 ) × = 9 ×
12 12 12
99 3
∴ S.D. of 3, 6, 9, 12, …..,30 = 9 × = 33
12 2
134. Ans. (b), If each term is multiplied by 10, then New S.D. = 10 × 2.71 = 27.1
135. Ans. (a), We have
Class Interval Frequency f i Mid-point xi f i xi xi − x (x − x)
2
f i ( xi − x )
2
i

4-8 3 6 18 −7 49 147
8-12 6 10 60 −3 9 54
12-16 4 14 56 1 1 4
16-20 7 18 126 5 25 175
Total 20 260 380

∑fx ∑ f (x − x)
2
260 380
∴ Mean ( x ) = i i
= = 13 ∴ Variance (σ 2 ) = i i
= = 19
∑f i 20 ∑f i 20
136. Ans. (b), Let A, the assumed mean, be 25.5. Here h = 10.
Classes xi x − 25.5 fi f i yi f i yi2
yi = i
10
0.5-10.5 5.5 −2 11 −22 44
10.5-20.5 15.5 −1 29 −29 29
20.5-30.5 25.5 0 18 0 0
30.5-40.5 35.5 1 4 4 4
40.5-50.5 45.5 2 5 10 20
50.5-60.5 55.5 3 3 9 27
Total 70 −28 124
 −28 
Mean ( x ) = 25.5 + (10 )   = 21.5
 70 

137. Ans. (d), Variance (σ 2 ) =


h2
N 2 { }
N ⋅ ∑ f i yi2 − ( ∑ f i yi ) =
2 10 × 10 
70 × 70 
2
70 (124 ) − ( −28 )  = 161.14

138. Ans. (a), S.D. (σ ) = 161.14 = 12.7 (approx)

th
22 Office.: 606 , 6 Floor, Hariom Tower, Circular Road, Ranchi-1, Ph.: 0651-2562523, 9835508812, 8507613968
( BY R. K. MALIK’S NEWTON CLASSES
) 23
139. Ans. (b), Let us take assumed mean = 155 and construct the following table.
xi fi x − 155 yi2 f i yi f i yi2
yi = i
5
140 4 −3 9 −12 36
145 6 −2 4 −12 24
150 15 −1 1 −15 15
155 30 0 0 0 0
160 36 1 1 36 36
165 24 2 4 48 96
170 8 3 9 24 72
175 2 4 16 8 32
Total 125 77 311
2
 f i yi2  ∑ f i yi  
∴ Variance = σ = h 

2 2
 25
 311  77 2 
−    = ×  −   = 52.7136
 N  N   
 125  125  

140. Ans. (a), S.D. = Variance = 52.7136 = 7.26.

141. Ans. (c), Given


∑x i
= 7 ⇒ ∑ xi = 18 × 7 = 126 Since an observation 12 was miscopied as 21,
18
correct ∑ xi 117
∴ Correct ∑x i = 126 − 21 + 12 = 117. Hence, correct mean =
18
=
18
= 6.5

142. Ans. (d), Also, variance = 4 2


= 16 ∴
∑ x − ( mean )
2
i 2
= 16 ⇒
∑x 2
i
= 16 + 7 2 = 6.5
18 18
⇒ Incorrect ∑x 2
i = 1170. ∴Correct ∑ x = 1170 − 21 + 122 = 873 .
2
i
2

correct ∑ xi2 2 873 2


Hence, correct variance = − ( correct mean ) = − ( 6.5 ) = 48.5 − 42.25 = 6.25
18 18
∴ Correct S.D. = correct variance = 6.25 = 2.5

143. Ans. (a), From given, we make the following table


x x2
2 4
3 9
11 121
λ λ2
∑ x = 16 + λ ∑x 2
= 134 + λ 2
2

But we know that, variance =


∑x 2
∑x
 ⇒
134 + λ 2  16 + λ 
−
2
49
(given)
−   =
n  n  4  4  4

134 + λ 2 ( 256 + λ + 32λ ) 49


2
3λ 2 − 32λ + 280 49
⇒ − = ⇒ = ⇒ 3λ 2 − 32λ + 280 = 196
4 16 4 16 4
14 14
⇒ 3λ 2 − 32λ + 84 = 0 ⇒ ( λ − 6 )( 3λ − 14 ) = 0 ⇒ λ = 6, λ = . Therefore, the value of λ are 6 and .
3 3

th
Office.: 606 , 6 Floor, Hariom Tower, Circular Road, Ranchi-1, Ph.: 0651-2562523, 9835508812, 8507613968 23
24 ( )BY R. K. MALIK’S NEWTON CLASSES
144. Ans. (a), To compare the variability, we have to calculate their coefficients of variation. Given,
variance of height = 127.69 cm 2 . Therefore, standard deviation of height = 127.69 = 11.3cm
Also, Variance of weight = 23.1361kg 2 . Therefore, standard deviation of weight = 23.1361 = 4.81 kg
standard Deviation
Now, the coefficient of variations (C.Vs.) are given by (C.V.) in height = × 100
Mean
11.3 4.81
= × 100 = 6.95 and (C.V.) in weights = × 100 = 9.18 . Clearly C.V. in weight is greater than
162.6 52.36
the C.V. in height. Therefore, we can say that weights show more variability than heights.
145. Ans. (a), The variance of the distribution of wages in plant A (σ 12 ) = 81 . Therefore, standard deviation
of the distribution of wages in plant A (σ 1 ) = 9 . Also, the variance of the distribution of wages in plant
B (σ 22 ) = 100 .Therefore, standard deviation of the distribution of wages in plant B (σ 2 ) = 10 . Since the
average monthly wages in both the plants is same, i.e., Rs. 2500, therefore, the plant with greater
standard deviation will have more variability. Thus, the plant B has greater variability in the individual
wages.
146. Ans. (a), Given, C.V .1 = 60, σ 1 = 21 C.V .2 = 70, σ 2 = 16 . Let x1 and x2 be the means of 1st and 2nd
σ 21 σ
distribution, respectively. Then C.V .1 = 1 × 100 ⇒ x1 = × 100 = 35 and C.V .2 = 2 × 100
x1 60 x2
16
⇒x = × 100 = 22.85
70
σ 12
147. Ans. (a), Here, n = 50. For Mathematics, C.V. = × 100 = × 100 = 28.57..... ( i )
x 42
σ 15
For Physics, C.V. = × 100 = 46.87 ..... ( ii )
× 100 =
x 32
σ 20
For Chemistry, C.V. = × 100 = × 100 = 48.89 ..... ( iii ) .From (i), (ii) and (iii), we have
x 40.9
C.V. of Chemistry > C.V. of Physics > C.V. of Mathematics ∴ Chemistry shows the highest
variability and Mathematics shows the least variability.
148. Ans. (b),
Size Mid Value xi fi x − 27.5 f i yi f i yi2
yi = i
5
10-15 12.5 2 −3 −6 18
15-20 17.5 8 −2 −16 32
20-25 22.5 20 −1 −20 20
25-30 27.5 35 0 0 0
30-35 32.5 20 1 20 20
35-40 37.5 15 2 30 60
100 8 150

Mean ( x ) = A +
∑ f y × h = 27.5 +
i i 8
× 5 = 27.5 + 0.4 = 27.9
N 100
h 5 1
N ∑ f i yi2 − ( ∑ f i yi ) ⇒ σ =
2 2
Standard deviation (σ ) = 100 × 150 − ( 8 ) = 15000 − 64
N 100 20
1 σ 6.11
= × 122.21 = 6.11 ∴ C.V. = × 100 = × 100 = 21.89.
20 x 27.9

th
24 Office.: 606 , 6 Floor, Hariom Tower, Circular Road, Ranchi-1, Ph.: 0651-2562523, 9835508812, 8507613968
( BY R. K. MALIK’S NEWTON CLASSES
) 25
149. Ans. (a), We are given that: n1 = 25, n2 = 35, x1 = 61, x2 = 58.
n1 x1 + n2 x2 ( 25 )( 61) + ( 35 )( 58 )
x= = = 59.25
n1 + n2 25 + 35
n1 x1 + n2 x2
150. Ans. (a), We are given that: x = 520, x2 = 420 and x = 500 , also we know x =
n1 + n2
⇒ 500 ( n1 + n2 ) = 520n1 + 420n2 ⇒ 20 n1 = 80 n2 ⇒ n1 : n2 = 4 :1
Hence the percentage of male employees in the firm = 80.
151. Ans. (c), Let the values of 9 items be x1 , x2 ........x9 . We have
x1 + x2 + ....x9
15 = ⇒ x1 + x2 + .... + x9 = 15 × 9 = 135.........(i )
9
x1 + x2 + .... + x9 + x10
Let x10 be the 10th item. The mean of x1 , x2 ,....x9 , x10 is 16. That is, 16 =
10
⇒ x1 + x2 + .... + x9 + x10 = 160 ⇒ 135 + x10 = 160 ⇒ x10 = 160 − 135 = 25.
152. Ans. (b),
Income (x) Frequency (f) (C.f.)
1000 14 14
1100 26 40
1200 21 61
1300 18 79
1400 28 107
1500 14 121

N + 1 121 + 1 N +1
Here = = 61 ∴ Median = Size of th item = 61 item = 1200.
2 2 2
153. Ans. (a), Given
Class group F c.f
110 – 120 6 6
120 – 130 25 31
130 – 140 48 79
140 – 150 72 151
150 – 160 (median class) 116 267
60 – 170 60 32
170 – 180 38 365
180 – 190 22 387
190 – 200 3 390
N 390
Median = Size of th item = size of = 195th item ∴ Median lies in the class 150 – 160
2 2
N /2−C
Median = L + × h , L = 150, N/2 = 195, C = 151, f = 116, h = 10
f
195 − 151
Median = 150 + × 10 = 150 + 3.79 = 153.79
116
154. Ans. (b),

th
Office.: 606 , 6 Floor, Hariom Tower, Circular Road, Ranchi-1, Ph.: 0651-2562523, 9835508812, 8507613968 25
26 ( BY R. K. MALIK’S NEWTON CLASSES
)
Wages/week (Rs.) No. of workers (f) c.f
50 – 59 15 15
60 – 69 40 55
70 – 79 50 105
80 – 89 (median class) 60 165
90 – 99 45 210
100 – 109 40 250
100 – 119 15 265
N 265
Median = size of   th item = = 132.5th item
2 2
Median lies in the class 80 – 89. But the real limit of the class is 79.5 – 89.5
Here, N = 265, C = 105, f = 60, h = 10 and l = 79.5
hN  10
Median = l +  − C  = 79.5 + (132.5 − 105 ) = 84.08 .
f 2  60
155. Ans. (d),We know that if y = x / h when σ y = σ x / h . ∴ the s.d. of new set of observations will be 4/4 = 1.
156. Ans. (d), As explained in the previous problem the s.d. of the new set of observation will be
8 / −2 = 4 .
157. Ans. (a), see theory
1/ n
158. Ans. (b), G = ( 7.7 2.73....7 n )
1/ n 1/ n n ( n +1) / 2
 = 7( n +1) / 2 > 7 n / 2.
= 71+ 2+......+ n  = 7

159. Ans. (c), Arranging data in ascending order. 1, 2, 3, 4, 5, 6, 7
th th
 n +1 7 +1  n +1
Q1 = Value of   observation = = 2nd observation, Q3 = Value of 3   observation
 4  4  4 
th
 7 +1  th Q3 − Q1
= 3  = 6 observation ∴ Semi-interquartile range = =2
 4  2
th th
  n + 1    n + 1 
160. Ans. (c), Upper Quartile = Size of 3    item = Size of 3    item … ∵∑ f i = 31
  4    4 
161. Ans. (c), Upper limit of the highest class (L) = 50, Lower limit of the lowest class (S) = 10
L − S 50 − 10
∴ Range = L – S = 50 – 10 = 40, Coefficient of range = = = 0.6
L + S 50 + 10
162. Ans. (c), (formula)
L − S 46 − 6
163. Ans. (a), Range = L – S = 46 – 6 = 40, Coefficient of range = = = 0.77
L + S 46 + 6
164. Ans. (b), Least possible value of x = Greatest Value – Range = 35 – 23 = 12
Q + Q3 Q − Q1
165. Ans. (d), 1 = 10 and 3 = 5 ⇒ Q1 + Q3 = 20 ..... ( i ) and Q3 − Q1 = 10 ...... ( ii )
2 2
Solving equation (i) and (ii), we get Q1 = 5 and Q3 = 15
L − S 100 − 50
166. Ans. (c), Range = L – S = 100 – 50 = 50, Coefficient of range = = = 0.33
L + S 100 + 50
Q3 − Q1 35 − Q1
167. Ans. (d), We have, = 10 and Q3 = 35 ⇒ = 10 ⇒ Q1 = 15
2 2

th
26 Office.: 606 , 6 Floor, Hariom Tower, Circular Road, Ranchi-1, Ph.: 0651-2562523, 9835508812, 8507613968
(BY R. K. MALIK’S NEWTON CLASSES
) 27
1 49 1 n 18
168. Ans. (b), Here, x = ∑ xi = = 7 ∴ M.D. ( x ) = ∑ xi − x = = 2.57
n 7 n i =1 7
169. Ans. (b), Arranging data in ascending order 50, 210, 240, 300, 310, 320, 340 ⇒ Md = 300
1 370
∴ M.D. ( Md ) =
n
∑ xi − 300 =
7
= 52.8

170. Ans. (c), (formula)


171. Ans. (b), (standard result)
172. Ans. (a), It is standard result that S.D. for first n natural number is
n2 − 1 n2 − 1 62 − 1 35
∴ S.D. = = =
12 12 12 12
173. Ans. (b), (See theory)
174. Ans. (c), As we know that S.D. is independent of change of origin. Hence S.D. will remain same.
175. Ans. (c), (Standard result)
X 1
176. Ans. (d), If X and Y are two variables such that Y = ( a ≠ 0 ) , then σ y = σ x
a a
8
∴ S.D. of the new observations = = 4,
−2
3 3
177. Ans. (a), S.D. = ( Q.D.) = ×16 = 24 (Standard result)
2 2
178. Ans. (c), As S.D. is independent of change of origin. Hence S.D. of new data is same

∑(x − x )
2
530 70
∑x ∑(x − x )
2 i
179. Ans. (c), x = = 53, i = 530, i = 70 ∴ S.D. = σ = = = 7 = 2.64
10 n 10
σ 2.64
∴ CV = × 100 = × 100 = 4.98 %
x 53
σ 7
180. Ans. (c), CV = × 100 = × 100 = 25 %
x 28
σ
181. Ans. (d), × 100 = 16 and x = 25 ⇒ σ = 4 ⇒ σ 2 = 16
x
S .D. 19.76
182. Ans. (d), CV = × 100 = × 100 %
Mean 35.16
S .D. 20
183. Ans. (a), We have, CV = 50 and S.D. = 20 ∴ CV = × 100 ∴ 50 = × 100
Mean Mean
20
∴ Mean = × 100 ∴ Mean = 40
50
184. Ans. (d), (See theory)
th
 9 +1  th
185. Ans. (d), Since n= 9, then median term =   = 5 term. Now, last four observations are
 2 
th
increased by 2.∵ The median is 5 observation, which is remaining unchanged. ∴ There will be no
change in median
186. Ans. (a), Arranging data in increasing order. 3, 3, 4, 5, 7, 9, 10, 12, 18, 19, 21 ⇒ n = 11

th
Office.: 606 , 6 Floor, Hariom Tower, Circular Road, Ranchi-1, Ph.: 0651-2562523, 9835508812, 8507613968 27
28 ( BY R. K. MALIK’S NEWTON CLASSES
)
th th
 n +1 rd  n +1 th
∴ Q1 =   value = 3 value = 4, and Q3 = 3   value = 9 value = 18
 4   4 
18 − 4 14
∴ Q.D. = = =7
2 2
b−a 1
187. Ans. (b), Let the lowest and highest scores be a and b respectively. ∴ b − a = 16 and =
b+a 3
⇒ b + a = 48
188. Ans. (a), Arranging the wages in ascending order. 7, 10, 12, 15, 17, 17, 25
th th
 n +1  n +1
∴Q1 = Value of   observation = 10, Q3 = Value of 3   observation = 17
 4   4 
1
∴ Q.D. = ( Q3 − Q1 ) = 3.5
2
189. Ans. (d), Upper limit of the highest class (L) = 60, Lower limit of the lowest class (S) = 35
L − S 60 − 35
∴ Coefficient of range = = = 0.26
L + S 60 + 35
Q − Q1 40 − 20 20
190. Ans. (d), We have, Q1 = 20 and Q3 = 40 ∴ Q.D. = 3 = = = 10
2 2 2

191. Ans. (a), Here, N = ∑ f i = 30, ∑ f i xi = 425, x =


∑fx i i
=
85
, ∑f xi − x = 150
i
N 6
1 150
∴ M.D. ( x ) =
N
∑f i xi − x =
30
=5

192. Ans. (b), N = ∑ f i = 44, ∑ f i xi = 660, x =


∑fx i i
=
660
= 15 and ∑f xi − 15 = 312
i
N 44
1 312
∴ M.D. ( x ) =
N
∑ fi xi − 15 =
44
= 7.09 ≅ 7

193. Ans. (c), Arranging data in ascending order 20,33, 39, 40, 50,53,59, 65, 69 ∴ Md = 50
1 114
∴ M.D(M) =
n
∑ xi − 50 =
9
= 12.67
n
f i xi n
194. Ans. (c), N = ∑ f i = 40, x = ∑ = 15 ∴ ∑ f i xi − x = 230
i =1 N i =1

1 n
230
∴ M.D. ( x ) =
N
∑f i =1
i xi − x =
40
= 5.75

195. Ans. (c), ∑ f ( y − µ ) = ∑ f y − µ∑ f = ∑ f y − ∑ f y



= 0 ..... ∵ µ =
∑fy i i


i i i i i i i i i  ∑f
 i 
196. Ans. (b), (See theory)
197. Ans. (c), (See theory)
1+ 2 + 6 + x + y
198. Ans. (b), Let the two unknown items be x and y, then x = 4.4 ⇒ = 4.4 ⇒ x + y = 13
5
12 + 2 2 + 6 2 + x 2 + y 2 2
Here, variance = 8.24 ∴ − ( mean ) = 8.24 ⇒ x 2 + y 2 = 97
5
Solving, x= 9, y= 4 or x = 4, y = 9
199. Ans. (a), S.D. will remain same as it is independent of change of origin

th
28 Office.: 606 , 6 Floor, Hariom Tower, Circular Road, Ranchi-1, Ph.: 0651-2562523, 9835508812, 8507613968
BY R. K. MALIK’S NEWTON CLASSES
( ) 29
5 5 3 3
200. Ans. (a), We know that, Q.D. = × M .D. = × 12 = 10 ∴ S.D. = × Q.D. = × 10 ⇒ S.D. = 15
6 6 2 2
201. Ans. (d), We know that when each item of a data is multiplied by λ , variance is multiplied by λ 2 .
Hence, new variance = 52 × 9 = 225
202. Ans. (c), Here, N = ∑ f i = 12, ∑fx i i = 132, ∑fx 2
i i = 1692
2
1692  132 
∴ Variance = −  = 141 − 121 = 20
12  12 
1 100 2

2
203. Ans. (c), Let x1 , x2 , x3 ,.....x100 be 100 observation. Then, Variance = xi − ( Mean )
100 i =1
1 100 2 n
∴ 25 = ∑ xi − 2500 ⇒
100 i =1
∑x
i =1
2
i = 252500

204. Ans. (d), We know that V ( aX ) = a 2V ( X ) ∴ Required variance = 2 2 × 23.33 = 93.32


n1 x + n2 x2
205. Ans. (d), σ 12 = 4, σ 22 = 5, x1 = 2, x2 = 4 and n1 = n2 = 5 ∴ x = = 3 , d1 = x1 − x = 2 − 3 = −1
n1 + n2

2
n1 (σ 12 + d12 ) + n2 (σ 22 + d 22 ) 11
d 2 = x2 − x = 4 − 3 = 1 ∴ σ = =
n1 + n2 2
206. Ans. (d), For finding the S.D. of 20, 22, 24, 26, 28, 30 first we subtract 18 from all the number we get
the data as 2, 4, 6, 8, 10, 12 and Now divide the data by 2 we get 1, 2, 3, 4, 5, 6, we know that S.D. of
n2 − 1 36 − 1 35
first n natural numbers is Hence S.D. of 1, 2, 3, 4, 5 and 6 is =
12 12 12
35 35
∴ S.D. of 2, 4, 6, 8, 10, 12 and 20, 22, 24, 26, 28, 30 is 2 = = 3.42
12 3
207. Ans. (a), It is given that each of the two population has 100 observations which are 100 consecutive
integers. So, sum of the squares of deviations from their respective means are same.
V
∴ VA = VB ⇒ A = 1 Note that S.D. of (1, 2, 3…100),(101, 102, 103…200) and(151,152,153,…250)
VB
will be same.
7990
208. Ans. (b), Corrected ∑ x = 40 × 200 − 50 + 40 = 7990 ∴ Corrected x=
200
= 39.95

Incorrect ∑ x 2 = n ∑ σ 2 + x 2  = 200 152 + 402  = 365000

364100
Corrected ∑ x 2 = 365000 − 2500 + 1600 = 364100 ∴ Corrected σ =
2
− ( 39.95) = 14.98
200
209. Ans. (a), Let the standard deviation of two distributions be σ 1 and σ 2 respectively. Then,
σ1
50 = ×100
30
σ2
and 60 = × 100 ∴ σ 1 = 15 and σ 2 = 15 ⇒ σ 1 − σ 2 = 0
25
th th
 n +1   21 
210. Ans. (d), N = ( ∑ f ) = 20 Now, Q1 =   observation =   observation = 3
 4   4

th
Office.: 606 , 6 Floor, Hariom Tower, Circular Road, Ranchi-1, Ph.: 0651-2562523, 9835508812, 8507613968 29
30 ( ) BY R. K. MALIK’S NEWTON CLASSES
x f cf
2 3 3
3 4 7
4 8 15
5 4 19
6 1 20
th th
 n +1  63  1 1
and Q3 = 3   observation =   observation = 5 ∴ Q.D. = ( Q3 − Q1 ) = ( 5 − 3) = 1
 4   4  2 2
211. Ans. (d), The given data in ascending order of magnitude is 7, 10, 12, 15, 17, 19, 25
th
 n +1 nd  3 ( n + 1) 
Here, Q1 = size of   item = size of 2 item = 10, Q3 = size of   item
 4   4 
Q − Q1 19 − 10
= size of 6th item = 19 Then, Q.D. = 3 = = 4.5
2 2
212. Ans. (a), Arrange the given data in ascending order, We hae 34, 38, 42, 44, 46, 48, 54, 55, 63, 70
46 + 48
Here, median = M = = 47 ….. (∵ n = 10, median is the mean of 5th and 6 th items )
2

∴ Mean deviation =
∑ x − M = ∑ x − 47
i i
=
13 + 9 + 5 + 3 + 1 + 1 + 7 + 8 + 16 + 23
= 8.6
n 10 10
213. Ans. (d), (standard result)
214. Ans. (a),We know that if each item of a data is increased or decreased by the same constant, the
standard deviation of the data remains unchanged. Hence new standard deviation = 40
215. Ans (d), The cumulative frequency curve of statistical data is called ogive.
216. Ans (c), The given observation
3times 5times 7times 5times 2times 1times
20, 20, 20 21, 21, 21, 21, 21 22, 22, 22, 22, 22, 22, 22 23, 23, 23, 23, 23 24, 24 25
Mode = Highest frequency of observation = 22 (7 times)
217. Ans (d),
x f fx
1 5 5
2 4 8
3 6 18
4 λ 4λ
∑f = 15 + λ ∑ fx = 31 + 4λ
∴ Mean =
∑ fx = 31 + 4λ ⇒ 3=
31 + 4λ
⇒ 45 − 31 = λ ⇒ λ = 14
∑ f 15 + λ 15 + λ
02 + 12 + 22 + 32 + ..... + n 2 n ( n + 1)( 2n + 1) 1
218. Ans. (b), Mean = = = n ( 2n + 1)
( n + 1) 6 ( n + 1) 6
1/ 3 1/ 3
219. Ans. (b), The geometric mean of the given observation = (10 × 40 × 60 ) = ( 24 × 1000 )
1/ 3 1/ 3 1/ 3
= ( 3) ( 8 ×1000 ) = ( 3) ( 2 ×10 ) = 20 3 3
220. Ans. (a), Firstly, arrange the given observation in ascending order 2, 3, 4, 5, 6, 7 and 9. Total number
of

th
30 Office.: 606 , 6 Floor, Hariom Tower, Circular Road, Ranchi-1, Ph.: 0651-2562523, 9835508812, 8507613968
( ) BY R. K. MALIK’S NEWTON CLASSES 31

terms, n = 7 (odd) So, Median = 


 n +1 ( 7 + 1) th term = 4th term = 5
th

 term =
 2  2
221. Ans. (d), We know that if each item of a data is increased or decreased by the same constant, the
standard deviation of the data remains unchanged. i.e., SD is 6.
222. Ans. (b), The given observations are arranged in ascending order 2, 2, 3, 3, 4, 5, 7, 9, 10.
th th
 9 +1   10  th
Here, total number of terms = 9 (odd) ∴ Median =   term =   term = 5 term = 4
 2   2
= ( 21+ 2 +3+ 4 +5 ) = ( 215 )
1/ 5 1/ 5 1/ 5
223. Ans. (c), Required geometric mean = ( 2 ⋅ 4 ⋅ 8 ⋅16 ⋅ 32 ) = 23 = 8
2 + 9 + 9 + 3 + 6 + 9 + 4 42
224. Ans. (b), Mean = = =6= X
7 7

∴ Mean deviation =
∑  X − X  = 2−6 +3 9−6 + 3−6 + 6−6 + 4−6
n 7
4 + 9 + 3 + 0 + 2 18
= = = 2.57
7 7
25 1
225. Ans. (d), From the pie chart, The share of industries = 25% = = . Let θ be the angle
100 4
1
corresponding to industries. Then, θ = × 360 = 90°
4
226. Ans. (c), We know that the arithmetic mean of n natural number with weights being the number itself
n ( n + 1)( 2n + 1)
∑ n2 = 6 =
n ( n + 1)( 2n + 1)
×
2
=
( 2n + 1) . For 16 natural numbers, put n = 16
∑n n ( n + 1) 6 n ( n + 1) 3
2
2 × 16 + 1 33
= = = 11
3 3
227. Ans. (c), Given, the marks of 10 students out of 15 in the ascending order are 40, 50, 60, 70, 70, 75,
th th
 n +1  15 + 1 
80, 80, 90, 95 ∴ Median = value of   term = value of   term = value of 8 term = 60
 2   2 
(Note that first five students are failed who have marks less than 40)
36 × 4 + 64 × 3 144 + 192 336
228. Ans. (c), Required mean = = = = 3.36
36 + 64 100 100
229. Ans. (c), Let the two other numbers be x1 and x2 . When, we take 12 wrongly in place of 8, then
geometric mean = 6
1/ 3
⇒ ( x1 ⋅ x2 ⋅12 ) = 6 ⇒ x1 ⋅ x2 ⋅12 = 216 ⇒ x1 ⋅ x2 = 18 ........ ( i )
Now, we take the right observation 8 in place of 12, then the geometric mean
1/ 3 1/ 3
= ( x1 ⋅ x2 ⋅ 8 ) = (18 ⋅ 8 ) = 2 3 18
230. Ans. (c), Given, HM = 21.6 and a = 27. Let the other number is ‘b’. We know that,
2ab 2 × 27 × b 583.2
HM = ⇒ 21.6 = ⇒ 583.2 = 54b − 21.6b ⇒ b = = 18
a+b 27 + b 32.4
7 + 9 + 11 + 13 + 15 55
231. Ans (d), X = = = 11 .
5 5

th
Office.: 606 , 6 Floor, Hariom Tower, Circular Road, Ranchi-1, Ph.: 0651-2562523, 9835508812, 8507613968 31
32 ( ) BY R. K. MALIK’S NEWTON CLASSES
2 2 2 2 2

Now, SD =
( 7 − 11) + ( 9 − 11) + (11 − 11) + (13 − 11) + (15 − 11) 16 + 4 + 0 + 4 + 16
5 5
= 8 = 2.8 (approx)
1 1 1
232. Ans. (d), Harmonic mean = = =
11 1 1  11 1 0
 + +   − 
3  3 ( −6 ) ( −6 )  3 3 3

20 × 15 + x
233. Ans. (d), Let the teacher’s age is x yr. 15 + 1 = ⇒ 16 × 21 = 300 + x ⇒ x = 336 − 300 = 36 yr.
21
234. Ans. (b), The largest amount occupies the largest area. Thus, the salaries occupied the largest area
6
∴ Sectorial angle = × 360° = 108°
20
1 n 1 n  xi 
235. Ans. (c), Let x1 , x2 , x3 ,....xn be n observation. Then, x = ∑
n i =1
xi ∴ New mean, x = ∑  + 10 
n i =1  α 
11 n  1 1 x + 10α
=  ∑ xi  + ⋅ (10n ) = x + 10 =
α  n i =1  n α α
236. Ans. (a), The sum of all the observations
2m + 1
S = a + ( a + d ) + ( a + 2d ) + ..... + ( a + 2md ) = [ 2a + 2md ] = ( 2m + 1)( a + md )
2
( 2m + 1)( a + md ) = a + md
∴ Required arithmetic mean = ( )
( 2m + 1)
x1 + x2 + x3 + ..... + xn
237. Ans. (d), Since, x is the mean of n observations x1 , x2 ,.....xn ∴ x =
n
x1 + ( a − b ) + x2 + ( a − b ) + ... + xn + ( a − b )
Now, ( a − b ) is added to each term. ∴ New mean =
n
x1 + x2 + .....xn n ( a − b )
= + = x + ( a − b)
n n
th
 5 +1
238. Ans. (b), Here, number of terms = 5 (which is odd) ∴ Median = value of   term
 2 
x x x x x
= value of 3rd term. Now, arranging the terms in increasing order,i.e., , , , , x the third term =
5 4 3 2 3
x
Given, median = 8 ⇒ = 8 ⇒ x = 24
3
SD
239. Ans. (d), Since, We know that Coefficient of variance = .
Mean
12 3.46
Coefficient of variance of Product A = = = 0.057
60 60
25 5
Coefficient of variance of Product B = = = 0.055 .
90 90
36 6
Coefficient of variance of Product C = = = 0.075
80 80

th
32 Office.: 606 , 6 Floor, Hariom Tower, Circular Road, Ranchi-1, Ph.: 0651-2562523, 9835508812, 8507613968
( ) BY R. K. MALIK’S NEWTON CLASSES 33

16 4
Coefficient of variance of Product D = = = 0.033
120 120
Hence, we see that minimum coefficient of variance is for Product D, hence product D is consistent.
7

∑x i 7
240. Ans. (c), Given, mean of 7 observations = 10 ⇒ i =1

7
= 10 ⇒ ∑x
i =1
i = 70 ..... ( i )

∑y i 3
and the mean of 3 observations = 5 ⇒ i =1
= 5 ⇒ ∑ yi = 15 ..... ( ii ) On adding Eqs. (i) and (ii),
3 i =1
7 3
we get ∑ xi + ∑ yi = 70 + 15 ⇒ Sum of all the ten observations = 85
i =1 i =1
10

∑x i
85
∴ Mean of 10 observations = i =1
= = 8.5
10 10
1/3
241. Ans. (c) G.M = ( 3.9.27 ) =9
3 60
242. Ans. (b) The harmonic mean of 2,4 and 5 is = = = 3.16
1 1 1 19
+ +
2 4 5
243. Ans. (d) N = ( ∑ f ) = 20
th th th
 N +1  21   N +1
Q1 =   observation =   observation = 3 , and similarly , Q3 = 3   observation
 4   4  4 
th
 63  1 1
=   observation = 5. Now Q.D . = ( Q3 − Q1 ) = ( 5 − 3) = 1
 4  2 2
x f c.f
2 3 3
3 4 7
4 8 15
5 4 19
6 1 20

True/ False
244. Ans. (T), (See Theory) 245.Ans. (T), (See Theory)
246. Ans. (F), (See Theory) 247.Ans. (), We know that A.M. ≥ G.M.
1/ 2
1 σ σ   σ σy  1
⇒  r x + r y  ≥  r x r. y
2  σ y σ x   σ y σ xx
 ⇒
2
( bxy + byx ) ≥ r.

Column Matching
248. Ans. (a), A.M. = 3 − 2 x = 3 − 2 (15) = −27 and S.D. = −2 σ = ( 2 )( 3) = 6 .

th
Office.: 606 , 6 Floor, Hariom Tower, Circular Road, Ranchi-1, Ph.: 0651-2562523, 9835508812, 8507613968 33

Vous aimerez peut-être aussi